Безопасное напряжение для человека: Безопасное напряжение — OUTDOOR-LIGHTING (Садовые светильники)

Содержание

Какой ток опаснее постоянный или переменный для человека — AC

Во всех современных квартирах и частных домах есть электроприборы и электропроводка, поэтому у жильцов всегда есть опасность попадания человека под высокое напряжение. Со времён Эдисона и Теслы в быту используется переменный ток, частота которого в Европе и странах СНГ составляет 50Гц.

Это значит, что его положительный и отрицательный вывода в розетке меняются местами 100 раз в секунду. Необходимое для работы многих устройств постоянное пониженное напряжение вырабатывается во встроенных или внешних блоках питания.

Несмотря на кажущуюся безопасность бытового напряжения, при неосторожном обращении с ним можно получить серьёзные электротравмы, вплоть до летального исхода. В этой статье рассказывается, каковы последствия воздействия электроэнергии на организм, а также какой ток опаснее — постоянный или переменный.

Отличие между постоянным и переменным током

Согласно школьному курсу физики, электрический ток — это направленное движение заряженных частиц. В металлических проводах это свободные электроны, а в организме человека, на 55-70% состоящем из воды, эту роль выполняют ионы различных элементов, в основном поваренной соли.

Есть два типа электрического тока:

  1. Переменный ток (от англ. Alternative Current — AC). Направление движения электронов в проводах и полярность на выводах периодически меняется.
  2. Постоянный ток (от англ. Direct Current — DC). Его особенность в том, что полярность и направление движения электронов остаются неизменными.

В начале ХХ века было много споров между сторонниками постоянного и переменного напряжения. Победила концепция переменного тока. Его легче передавать от электростанции к потребителю, изменять величину напряжения, а электродвигатели переменного тока устроены проще и дешевле.

Несмотря на то, что величина переменного напряжение частотой 50Гц поднимается до максимума и снижается до «0» 100 раз в секунду, лампы накаливания светят ровным светом.

Это связано с тем, что такую частоту мерцания глаза не замечают, а нагретая нить накала не успевает остыть, что уменьшает их амплитуду. В светодиодных и энергосберегающих лампочках питание осуществляется от встроенного драйвера (блока питания), который исключает колебания светимости.

Постоянный ток: безопасность при низком напряжении

Электрическое напряжение любого типа является безопасным только до определённого уровня. Например, вывода плоской батарейки 3R12 4,5В при прикосновении к ним языка имеют кислый вкус, «Крона» 9В вызывает болезненные ощущения, а вывода из прикуривателя автомобиля пробовать языком вообще не следует, но совсем не ощущаются сухими руками без порезов.

Аппарат электрофореза, который используется в поликлиниках, имеет выходное напряжение =60В, что приводит к протеканию тока через организм 50мА. Это воздействие вызывает лёгкое пощипывание. Такое же переменное напряжение, поданное через электроды и влажную марлю, может нарушить ритм сердечных сокращений. В связи с этим постоянный ток при напряжении до 48В считается безопаснее переменного.

Важно! Некоторые «мастера» при ремонте зарядок мобильных телефонов и планшетов проверяют исправность аппарата языком. Несмотря на низкое 5В напряжение, при некоторых неисправностях аппарата на выходе может оказаться потенциал 220В, поэтому такая проверка является смертельно опасной.

Постоянный ток: опасность при высоком напряжении

При величине напряжения выше 500В постоянный ток также опасен, как и переменный. Попадание человека под постоянное напряжение может вызвать нарушение работы сердца. Это относится не только к сети, но и к заряженным конденсаторам ёмкостью большой ёмкости.

Энергии, запасённой в элементе, достаточно для нанесения вреда здоровью. Сравнивая эти два вида напряжения можно сказать, что однозначного ответа, что опаснее — переменный или постоянный ток дать нельзя.

Это зависит от напряжения:

  • до 500 В переменный ток AC может причинить бОльший вред, чем постоянный;
  • при 500 В оба вида напряжений опасны примерно одинаково;
  • выше 500 В постоянный ток DC более опасен.
Важно! Полностью безопасным является только пониженное напряжение. Его величина зависит от типа помещений и указывается в ПТБ и ПУЭ.

Переменный ток: безопасность при высокой частоте

Есть разновидность переменного напряжения, которая является безопасной при любом потенциале. Это напряжение частотой более 20кГц. Благодаря скин-эффекту ток сверхвысокой частоты проходит по поверхности кожи, не попадая внутрь организма к жизненно важным органам. Никола Тесла при демонстрации этого явления прикасался к электродам, находящимся под напряжением 100кВ с частотой 100кГц без вреда для здоровья.

Какой ток опасен для человек

На самом деле само по себе напряжение опасным не является. Опасен ток, проходящий через организм человека. Он зависит не только от потенциала, но и от сопротивления кожных покровов в месте контакта и других факторов:

  • сухая обувь, чистые руки и деревянный пол уменьшают опасность поражения;
  • приём спиртных напитков перед работой также уменьшает сопротивление и увеличивает поражающую силу высокого напряжения.
Интересно! Несмотря на то, что напряжение на электродах ионизатора воздуха достигает 25 кВ, ток при прикосновении настолько мал, что не ощущается человеком.

При одинаковых условиях переменный ток считается более травмоопасным. Это связано с тем, что опасный ток для человека зависит от вида напряжения — безопасная величина переменного тока, на которую настраивается уставка УЗО, составляет 10 мА, а для постоянного опасное воздействие начинается с 50 мА, которые являются максимальными для работы аппарата электрофореза.

Кроме силы тока, проходящего через организм, степень воздействия зависит от пути его прохождения и продолжительности контакта пострадавшего с токоведущими частями.

И всё же, почему переменный ток опаснее постоянного? Есть несколько факторов, делающих его воздействие более опасным:

  • Постоянный ток при протекании через организм вызывает спазм мускулатуры. Это менее опасно, чем сокращение и расслабление мышц под воздействием переменного тока. Поэтому для причинения одинакового вреда здоровью величина постоянного тока и, соответственно, напряжение должны быть в несколько раз выше, чем переменного.
  • Электротравмы со смертельным исходом чаще всего происходят от фибрилляции желудочков сердца. Это состояние возникает от воздействия переменного напряжения и может потребовать реанимационных действий и использования дефибриллятора. При воздействии постоянного тока происходит спазм сердечной мышцы, который может пройти после освобождения человека от напряжения.
  • Есть широко распространённое мнение, что при попадании под переменное напряжение легче освободиться самостоятельно. Это связано с тем, что при таком воздействии происходит периодическое расслабление мускулатуры. Такая версия была бы правильной, если бы частота в розетке была 1-2 Гц, но при частоте 50Гц сокращения и расслабления происходят 100 раз в секунду и паузы между спазмами настолько короткие, что человек не успевает на них отреагировать.

Проведённые эксперименты подтверждают, что взятый в руку электрод с постоянным напряжением, получается отпустить легче и быстрее.

Как видно из материалов статьи, при одинаковом напряжении травмы при поражении переменным током опаснее, чем постоянным.

Важно! Полностью безопасного напряжения, кроме сверхнизкого, не существует. При работе необходимо соблюдать осторожность и все требования, указанные в ПТБ и ПТЭЭП.

Пути прохождения тока через тело человека

То, какой ток опасен для человека, зависит не только от его величины, но и от пути прохождения через тело

.

При попадании человека под напряжение ток стремится пройти по кратчайшему расстоянию. В зависимости от места контакта попасть в зону поражения могут различные органы и части тела. Есть различные варианты прохождения электрического тока через тело. Некоторые из них встречаются реже, другие чаще.

Особенно опасным являются те пути прохождения тока, при которых происходит поражение сердца, спинного и головного мозга и лёгких. Правда, это не значит, что остальные пути являются безопасными.

Информация! В статистику электротравматизма попали только такие случаи, при которых пострадавшенму потребовалась медицинская помощь.

Рука — рука

Чаще всего электромонтёры травмируются во время работы при прикосновении разными руками к фазному проводнику и к заземлённой конструкции или к другому фазному проводу.

Такие травмы составляют около 40% всех обращений к врачам. Ток идёт через верхнюю часть груди и до 3,3% проходит через сердце. При травмировании напряжением 220 В до 83% пострадавших теряют сознание.

Правая рука — ноги

Прохождение электрического тока по пути «рука-ноги» является опасным для жизни. Электроэнергия проходит через сердце, ноги и спинной мозг, причём на сердечную мышцу приходится 6,7%.

Такая электротравма происходит, если на работнике надета обувь с гвоздями в подошве, а пол бетонный или влажный деревянный.

Частота этих травм составляет 20%, количество потерявших сознание 87%.

Левая рука — ноги

Причины травмирования в этом случае аналогичны ситуации «правая рука — ноги», но встречается несколько реже — в 17% случаев. Это связано с тем, что большинство людей предпочитают работать правой рукой.

Доля тока, проходящего через сердце, составляет 3,7%, поэтому количество пострадавших потерявших сознание 80%.

Нога — нога

Такое прохождение тока происходит при попадании человека под шаговое напряжение. Этих случаев всего — 6%. Доля тока через сердце составляет 0,4%.

Основная опасность таких травм заключается в судорогах или спазмах ног. При этом человек может упасть и величина шагового напряжения увеличится, а ток пойдёт по пути «руки-ноги» или «голова-ноги», поэтому пострадавшие теряют сознание в 15% случаев.

Голова — ноги

Достаточно редкая, около 5% случаев, но опасная ситуация. Возникает при работе без головного убора в распредустройствах и высоких панелях управления.

Поражаются головной и спинной мозг, позвоночник и внутренние органы. Часть тока, проходящая через сердце 6,8%, До 88% пострадавших теряют сознание и нуждаются в реанимационных мероприятиях и госпитализации.

Голова — руки

Эта ситуация опаснее, чем травма «голова-ноги». Часть тока, проходящего через сердце, составляет 7%, попавшие под напряжение теряют сознание в 92% случаев.

Причины травмирования аналогичны предыдущей, частота появления составляет 4%.

Другие пути

Около 8% случаев электротравматизма связаны с прикосновением к токоведущим частям другими частями тела. Чаще всего это происходит при работе без спецовки или летом, в расстёгнутых куртках без рубашки.

Важно! Для предотвращения электротравматизма необходимо соблюдать требования ПТБ и использовать основные и дополнительные защитные средства — перчатки, галоши, коврики и инструмент с изолированными ручками.

Величина тока опасная для жизни человека в Амперах

В таблице можно увидеть, какой ток опаснее постоянный или переменный при разных значениях — от практически неощутимых до смертельно опасных.

Как видно из материалов статьи и таблицы, ответ на вопрос «какой ток опаснее постоянный или переменный» однозначный — переменное напряжение является значительно более опасным.

 Сила тока, мА  Воздействие на организм
Переменный ток  Постоянный ток 
 0,6-1,5 небольшой зуд и незначительное покалывание кожи  не ощущается 
2-3  при протекании через руку её сводит незначительной судорогой, может ощущаться не только кистью, но и запястьем  не ощущается 
5-7  сильные болезненные судороги  лёгкое покалывание и нагрев поверхности кожи 
 8-10 сильная боль и судороги, сохраняется возможность самостоятельно разжать руки и освободиться от воздействия тока  возрастают покалывание и нагрев кожи 
 20-25  паралич конечностей, значительно затрудняется дыхание, возможность отпустить оголённый провод усиливается нагрев и покалывание поверхности кожи, появляются незначительные судороги 
 50-80 паралич дыхательных мышц, нарушение сердечного ритма   значительный нагрев кожи, усиливаются судороги мышц, затрудняется дыхание
90-100  остановка дыхания, через 3 секунды начинается фибрилляция сердца, без реанимационных действий возможен летальный исход  паралич дыхательной мускулатуры 

С переменным напряжением современные люди сталкиваются всё время, дома и на производстве. Для того чтобы предотвратить электротравмы необходимо соблюдать технику безопасности при эксплуатации электроприборов. Особенно важно обучить правилам пользования электричеством детей.

Розетки в доме должны иметь заземляющий контакт, необходимо также во вводном щитке установить УЗО или дифференциальный автомат.

Похожие материалы на сайте:

Понравилась статья — поделись с друзьями!

 

Действие электрического тока на организм человека, причины электротравм

Проходя через тело человека, электрический ток может вызвать поражение внутренних или внешних органов.

При поражении внутренних органов может наступить паралич органов дыхания или фибриляция, что часто влечёт за собой смертельный исход.

При поражении внешних органов могут иметь место ожоги в результате прохождения через кожу человека значительных токов или в результате непосредственного воздействия электрической дуги.

Специфическая особенность проявления опасности электрического тока заключается в отсутствии каких-либо внешних признаков, предостерегающих человека об угрожающей ему опасности.

Рисунок 1. Электричество опасно, но не всегда.

Опасное действие электрического тока на организм человека зависит главным образом от величины тока, протекающего через тело человека, пути тока и длительности его воздействия.

Для разных людей и условий предельная величина опасного тока различна.

В среднем при длительном действии:

0,5 mA – ощущается человеком;

2 3 mA – появляется боль;

15 mA – резко выраженная судорога с трудно переносимой болью;

Степень воздействия эл. тока на человека в зависимости от последствий классифицируют:

Ощутимый ток – наименьшее значение тока, который ощущается человеком. 0,8 1,8 mA при переменном токе с частотой 50 Гц и 5 7 mA при постоянном токе. Но известны случаи, когда значительно меньшие токи повлекли смертельный исход.

Отпускающий ток – наибольшее (пороговое) значение тока, при котором человек сохраняет способность самостоятельно и произвольно освободиться от контакта с частями, находящимися под напряжением. 4 8 mA при f = 50 Гц, постоянный ток в 3,5 4 раза больше.

Не отпускающий ток – наименьшее значение тока, при котором человек теряет способность самостоятельно и произвольно освободиться от контакта с частями, находящимися под напряжением и, следовательно, подвергается смертельной опасности при длительном воздействии тока. Для переменного тока 50 Гц 8 16 mA, а постоянного тока — 50 80 mA.

Смертельный ток. Большинство специалистов оценивают этот ток на уровне 100 mA и более, однако исследования последних лет показывают, что порог смертельного тока может быть в 3 4 раза ниже.

Опасность поражения организма человека током зависит от продолжительности воздействия тока. Для определения предельного тока (допустимого) Международной электротехнической комиссией рекомендована формула:

Iдоп.=10+10/t mA

где t — длительность воздействия тока на человека, секунд. Формула справедлива при t > 0.1-0.2 с.

Для t<0.1 с.

Iдоп.=240/Vt, при t= 0.001-0.01 c.

Iдоп.=760/4Vt, при t= 0.01-0.1 c.  Предельно допустимые уровни напряжений прикосновений и токов при аварийном режиме производственных энергоустановок напряжением до и выше 1000В с глухо заземлённой или изолированной нейтралью регламентированы (в зависимости от t). Для электроустановок с изолированной нейтралью UДОП.ПР = 36В, IДОП = 6 mA при t>1 с и f = 50 Гц.

Кроме величины и длительности воздействия тока опасность поражения зависит также от: Пути тока; Рода и частоты тока;

Состояния организма и физиологических особенностей человека и ряда других второстепенных факторов;

Основными факторами, определяющими величину тока, проходящего через тело человека, являются сопротивление тела человека и величина приложенного к телу напряжения.

Сопротивление тела человека зависит от большого количества факторов:

  • места контакта,
  • размеров поверхности соприкосновения,
  • состояния кожи (толщина рогового слоя),
  • её влажности,
  • загрязнённости,
  • величины приложенного напряжения и протекающего тока, под действием которого сопротивление тела человека, обладающее нелинейностью, сильно меняется.

 При напряжении 20 30В сопротивление тела остаётся почти неизменным. С увеличением приложенного напряжения в пределах от 30 до 250В сопротивление тела резко уменьшается. При напряжении около 250В наступает резко выраженный электрический пробой кожи, при этом сопротивление снижается от нескольких десятков и даже сотен тысяч до 1000 Ом и ниже. При напряжении 40 45В и выше сопротивление тела человека уже мало зависит от состояния кожи и степени её увлажнённости.

В шахтных условиях, учитывая влажность, наличие токопроводящей пыли и повышенное потовыделение, следует принимать нижнюю границу сопротивления тела человека, т. е. 1000 Ом. По данным МГИ, при расчёте электроустановок карьеров на электробезопасность с учётом специфики условий труда и окружающей среды сопротивление тела человека следует принимать:

  • при напряжении U<1000В – 0,8 кОм,
  • при напряжении U>1000В – 0,5 кОм.

До последнего времени считалось, что наиболее опасен для человека эл. ток f = 50 60 Гц. Исследования показали, что с ростом частоты тока от 50 Гц до 15 кГц значения отпускающих токов возрастают за исключением частоты 200 Гц, которая может рассматриваться как наиболее физиологически активная. Зависимость отпускающих токов IОТП от f (в пределах 200 15000 Гц) выражается формулой:

Iотп.= k*Vf


где k 0,45 – коэффициент, зависящий от условий воздействия и площади контакта с токоведущими частями.

Статистические исследования электротравматизма в различных горнодобывающих отраслях выявили причинно-следственные связи с целым рядом факторов.

Величина рабочего напряжения

70 80% электротравм на карьерах произошли в электроустановках при U>1000В (6 кВ при переменном токе). Поэтому важнейшей проблемой остаётся борьба с однофазными замыканиями на землю в карьерных распределительных сетях U = 6 кВ.

Место происшествия и вид электрооборудования

Электротравмы на ВЛ, как правило, чаще, чем на КЛ. Поэтому важно разрабатывать рациональные схемы электроснабжения карьеров. Основное число электротравм приходится на персонал, обслуживающий РУ, ВЛ и КЛ 3 10 кВ, электрооборудование экскаваторов и электровозов, а также тяговые сети U>1000В. Значительное число несчастных случаев происходит при пусконаладочных и ремонтно-монтажных работах на РУ стационарных и передвижных подстанций, а также приключательных пунктах.

Профессии, возраст и производственный стаж пострадавших

Большинство электротравм приходится на электротехнический персонал при U>1000В. Более 50% пострадавших – работники в возрасте <32 лет. На работников со стажем работы <5 лет приходится >50% электротравм.

Время происшествия несчастных случаев

Для карьеров пики электротравматизма наступают в весенний, летний и осенний периоды (наибольший пик летом).

На уровень электротравм влияют факторы влияния и усталости. Наибольшее число электротравм происходит в часы смен, соответствующих началу и окончанию работ. Максимум электротравм приходится на первую (утреннюю) рабочую смену, когда выполняется наибольшее количество работ.

Основные причины электротравматизма.

1-я группа. 80 90% происходит в результате прикосновения к токоведущим частям электроустановок. Много травм при работах без снятия напряжения.

2-я группа. 20% электротравм при ошибочной подаче напряжения и неправильном отключении электроустановок. Прикосновение к нетоковедущим частям электроустановок, оказавшимся под напряжением. Замыкания на корпус электроустановок вследствие ухудшения состояния изоляции.

Электротравмы происходят по нескольким причинам: организационным, техническим, психофизиологическим.

К организационным причинам относят обычно электротравму, связанную с невыполнением ПБ. К техническим причинам относят электротравму, связанную со снижением уровня изоляции, механическим повреждением и т. п.

На карьерах наибольшее число травм (более 70%) происходят по организационным причинам.

Для расчётов, связанных с обеспечением защиты от поражения электрическим током людей, соприкасающихся с электроустановками, необходимо знать предельную величину длительного безопасного тока , а также предельно безопасную величину напряжения прикосновения UПР. Существующие ПУЭ и ПБ не регламентируют ни предельной безопасной величины тока, ни допустимой величины напряжения прикосновения.

Во Франции для шахт установлены следующие предельно безопасные величины тока: при постоянном токе 50 mA, при переменном токе промышленной частоты – 25 mA.

В Англии и ФРГ за безопасное значение переменного тока в шахтах принимают 50 mA.

В РФ для угольных шахт «Правила изготовления взрывозащищённого и рудничного электрооборудования» (ПИВРЭ) предписывают как предельно безопасную величину длительного тока 30 mA, а при автоматической компенсации ёмкостной составляющей тока утечки – 25 mA.

«Правила безопасности в угольных и сланцевых шахтах» не регламентируют величину безопасного тока и напряжения прикосновения, однако предписывают автоматическую защиту от утечек тока и прикосновений к токоведущим частям, продолжительность действия которой не должна превышать 0,2 с.

Сопоставим это требование с таблицей RЧ = 700 Ом, IК.Б. = 250 mA, UПР = 175В (t=0,2c).

Таким образом, в шахтных условиях (при наличии защиты от утечек) ток 30 mA длительностью не выше 0,2 с тем более можно считать безопасным для человека.

Поле напряжений безопасное — Энциклопедия по машиностроению XXL

Большую сложность представляет решение проблемы вредного воздействия электрического поля на человека. Проведены исследования, в результате которых установлены санитарно-гигиенические нормативы по напряженности электрического поля, обеспечивающие безопасность персонала, обслуживающего линии и подстанции, а также населения в зоне влияния воздушных линий. Исследования в этом направлении продолжаются.  [c.238]

Рассмотрим некоторые конкретные случаи выработок, для которых условие безопасности имеет особенно простой вид Вначале отметим, что для тяжелого однородного и изотропного упругого полупространства z поле напряжений вдали от выработки определяется следующими формулами  [c.216]


Теорема Мелана. Приспособляемость наступит, если можно найти такое не зависящее от времени поле фиктивных остаточных напряжений а,-у, что при любых изменениях нагрузки в заданных пределах сумма этого поля с полем напряжений Оц в идеально упругом теле безопасна (достаточное условие).  [c.338]

Электромагнитная муфта Y1 (рис. 465) питается постоянным током, напряжение которого по условиям техники безопасности не должно превышать 24 В. При напряжении сети переменного тока 380 В питание электромагнитной муфты YI осуществляется через однофазный трансформатор TI (с ферромагнитным сердечником) и выпрямительное устройство VI (выполненное с применением полу-  [c.278]

В настояш ее время, в связи с коренной перестройкой топливно-энергетической базы нашей страны в направлении резкого повышения роли ядерного горючего вместо природного газа, и, особенно, жидкого органического топлива, существенно возросла потребность в атомных энергетических установках. Организация их производства может быть основана на выпуске конструкций в многослойном исполнении, что в значительной степени будет способствовать решению всей проблемы. При этом, однако, следует иметь в виду, что атомные установки работают в более сложных и тяжелых условиях, чем сосуды химической промышленности и степень их ответственности значительно выше. Отсюда возникает необходимость в проведении комплекса работ, направленных на обеспечение надежности, долговечности п экономичности изготовления корпусов атомных реакторов, пароперегревателей, емкостей безопасности, защитных корпусов и др. Особое внимание должно быть обращено на вопросы, связанные с установлением напряженно-деформированного состояния многослойных стенок и сварных узлов конструкций, сопротивляемостью их хрупким и квазихрупким разрушениям, расчетами температурных полей в многослойных элементах, оценкой циклической прочности, изучением динамической и термоциклической стойкости конструкций, методам контроля, разработкой нормативных материалов по расчету на прочность.  [c.23]

Электромагнитная катушка рассчитана на создание магнитного поля номинальной напряженностью 150 ООО А/м. Для большей безопасности ЭМФ должен быть огражден на расстоянии не менее 0,5 м, при этом напряженность магнитного поля за ограждением будет меньше допустимой (8 кА/м). Устройства автоматики и питания со щита с электрическим напряжением 380 В устанавливаются на расстоянии не менее 5 м.  [c.100]


Для измерения лазерного излучения фотоэлементы применялись с самого начала [49] и теперь получили очень широкое распространение [50—52]. При количественных измерениях не следует забывать обычных мер предосторожности, т. е. не выходить за допустимые пределы пикового и среднего тока фотоэлемента, чтобы предотвратить явления усталости и положительной или отрицательной обратной связи, экранировать фотоэлементы от магнитных и электростатических полей и пользоваться для питания хорошо стабилизированными источниками напряжения, чтобы задавать усиление ФЭУ. Выходная мощность многих лазеров столь велика, что для уменьшения интенсивности пучка до безопасного уровня следует применять подходящие расщепители или ослабители пучка.  [c.121]

В сухих производственных помещениях относительно безопасным считается напряжение тока до 40 в. В помещениях жарких, сырых, помещениях с земляным или бетонным полом безопасно напряжение только до 12 в. Провода высокого напряжения должны быть размещены в местах, не доступных для соприкасания с ними.  [c.368]

Относительно безопасным следует считать напряжение тока до 40 в, в сухих производственных помещениях безопасным—напряжение до 36 в, а в помещениях особо опасных (в жарких, сы рых с земляным или бетонным полом, где человек имеет хороший контакт с землей) безопасным — напряжение до 12 в.  [c.280]

Достоинство электростатического напыления заключается в том, что из-за эффекта огибания электростатическим полем детали и ее отдельных выступов удается получить качественное покрытие на фланцах, сварных швах и других выступающих частях изделия. При это.м методе не требуется предварительного нагрева изделия. Коэффициент использования материала достигает 90%, что существенно, например, при нанесении покрытий из дорогостоящих фторполимеров. В качестве источника тока высокого напряжения применяют любой генератор, дающий ток силой до 10 мА при напряжении 80—150 кВ, что обеспечивает безопасность работы. Покрытие можно наносить как ручным способом пистолетом-распылителем, так и на установках, полностью механизированных и автоматизированных.  [c.258]

Правила техники безопасности при эксплуатации сварочных трансформаторов. В процессе работы электросварщик постоянно обращается с электрическим током, поэтому все-токоведущие части сварочной цепи должны быть надежно изолированы. Ток величиной 0,1 а и выше опасен для жизни и может привести к трагическому исходу. Опасность поражения электрическим током зависит от многих факторов и в первую очередь от сопротивления цепи, состояния организма человека, влажности и температуры окружающей атмосферы, напряжения между точками соприкосновения и от материала пола, на котором стоит человек.  [c.19]

Для нормального и безопасного обслуживания оборудования котельное помещение должно быть в достаточной мере освещено днем естественным светом, а в темное вре-ся суток — электрическим. Электрические лампы общего и местного освещения, подвешиваемые ниже 2,5 м от уровня пола или площадок, подключают к сети напряжением не более 36 В. Напряжение 127 и 220 В разрешается применять лишь для специальных светильников, лампы в которых заменяют только электромонтеры.  [c.113]

Для того чтобы сделать прессовочный порошок более компактным и удобным в работе, его нередко перед закладыванием в пресс-форму таблетируют, т. е. превращают небольшим давлением в маленькие заготовки—т а б-летки. Для ускорения технологического процесса горячего прессования и повышения качества прессуемых изделий целесообразно перед загрузкой в пресс-форму материал подогревать. Наиболее совершенный и высокопроизводительный способ предварительного подогрева таблеток — подогрев за счет выделения в них тепла диэлектрических потерь при помещении в электрическое поле высокой частоты (от 5 до 50 Мгц). Для этой цели применяются специальные устройства, включающие ламповый высокочастотный генератор и воздушный конденсатор, между пластинами которого помещаются подлежащие обогреву таблетки устройство оформляется в виде шкафа, причем конденсатор для безопасности работающих помещается в ящике, дверца которого снабжается блокировкой, обеспечивающей снятие напряжения с электродов при открывании дверцы. При высокочастотном нагреве тепло выделяется во всей толще нагреваемого материала, а не подводится извне, как при обычных способах нагрева (в термостатах и пр.), что обеспечивает быстроту и равномерность нагрева так, высокочастотный генератор мощностью 1 тт дает возможность нагрева 1 кг пресс-материала до 120—130° С примерно за 2 мин.  [c.125]

Величина безопасного напряжения устанавливается в зависимости от характера помещения, в котором оно используется. В сырых (влажность от 75 до 100%) помещениях и особо сырых (постоянная влажность 100% пол, потолок, стены покрыты влагой) опасность поражения током резко возрастает. Поэтому в них безопасным считается напряжение 12 В.[c.29]

В зданиях, где установлены однобалочные и двухбалочные подвесные краны, не имеющие галерей и площадок для обслуживания механизмов (ст. 226), должны быть устроены ремонтные площадки, позволяющие иметь удобный и безопасный доступ к механизмам и электрооборудованию. При расстоянии от пола ремонтной площадки до нижних частей крана менее 1800 мм дверь для входа на ремонтную площадку должна быть оборудована запором и автоматической электроблокировкой, снимающей напряжение с главных троллейных проводов ремонтного участка. Вместо устройства стационарных ремонтных площадок допускается применение передвижных площадок.  [c.33]

При опасных условиях работы (в сырых или жарких помещениях с хорошо проводящим ток полом) безопасным считается напряжение не выше 12 в.  [c.149]

Техника безопасности при окрашивании в электрическом поле. В связи с тем, что этот метод окрашивания деталей связан с применением высокого напряжения (порядка 100 кв) для обеспечения безопасности при работе необходимо соблюдать определенные меры предосторожности.  [c.165]

Для осмотра автомобилей применяют переносные безопасные электролампы напряжением до 36 в с предохранительными сетками, при работе в осмотровых канавах напряжение не должно превышать 12 в. Ручные электроинструменты (дрели, гайковерты) надо присоединять к сети только через штепсельные розетки с заземляющим контактом. Провода электроинструментов нужно подвешивать, не допуская соприкосновения их с полом.  [c.325]

Особая опасность литейных цехов в отношении поражения электрическим током обусловлена наличием большого количества металла, высокой температурой, токопроводящими полами, металлической пылью, наличием сырости. Поэтому устройство и эксплуатация электрических установок в литейных цехах должны отвечать требованиям утвержденных Министерством электростанций СССР Правил устройства электротехнических установок промышленных предприятий и Правил техники безопасности при эксплуатации электротехнических установок промышленных предприятий . Всякая новая установка и установка, подвергшаяся ремонту, может быть принята в эксплуатацию только при удовлетворительных результатах измерения сопротивления изоляции, т. е. сопротивление изоляции ( Яиз) на любом участке сети между двумя предохранителями при рабочем напряжении ( ) должно быть не менее > 1000 и.  [c.577]

При изменении полярности 100 раз в секунду ток и напряжение на дуге проходят через нулевое значение дуга в это время гаснет. Для надежного восстановления дуги на обратной полярности источнику питания необходимо иметь напряжение холостого хода свыше 200 в. Такое напряжение не может быть применено по условиям техники безопасности и экономическим соображениям. На практике сварочную дугу питают от трансформатора с напряжением холостого хода около 60 в. Зажигание и восстановление горения дуги осуществляют при помощи посторонних ионизаторов, включенных в схему осцилляторов или генераторов импульсов. Осцилляторы на сварочную дугу подают переменное напряжение от 3000 до 6000 в, которое является безопасным, так как его частота составляет 150—500 кгц. Осцилляторы позволяют зажигать дугу, не касаясь непосредственно электродом изделия. Генераторы импульсов подают на дугу строго синхронно (в начале полу-периода обратной полярности) импульсы напряжения величиной около 300 в, обеспечивающие стабильное горение дуги, обычно возбужденной при помощи осциллятора.  [c.86]

Напряжение 12 В считается безопасным напряжение холостого хода источников питания, допускаемое ГОСТ 95—69 и ГОСТ 304—69 (80 В для переменного и 90 В для постоянного тока), представляет опасность при определенных условиях (металлический пол рабочего помещения, большая влажность, работа при атмосферных осадках и т. п.). Напряжение холостого хода источников питания для плазменно-дуговой резки —до 500 В (ГОСТ 14935—69) представляет прямую опасность для операторов.  [c.183]

По соображениям безопасности лучше применять для нагрева ванн ток напряжением ПО в вместо 220 в с защитным заземлением 19]. Реостаты, регулирующие нагрев, следует располагать на отдельных щитах на расстоянии не менее 0,5 м от ванны и не менее 1 м от пола рукоятки реостатов, переключающих сопротивления, должны быть расположены на расстоянии не менее 1,3—1,5 м от уровня пола [19],  [c.31]

ВИДЫ оружия анализируют лишь номинально, причем особое значение придают использованию опытных коэффициентов безопасности, а также проведению испытаний прототипа на выносливость. При проектировании других видов оружия проводят детальный расчет на основе теоретических и экспериментальных данных, чтобы получить совершенную конструкцию прототипа для испытания ее на выносливость. Руководяш,ие материалы по усталостной прочности отражают обилий уровень знаний в области усталостного разрушения. В настоящее время еш е остаются вопросы теоретические и феноменологические, для решения которых недостаточно знаний, например, о влиянии на усталость материала таких факторов, как поле напряжений, остаточные напряжения, масштабный фактор, обработка и состояние поверхности, а также качество материала. Последний обзор теоретических положений и методов, относяш ихся к накапливаемому повреждению (Хардат,  [c.319]

Дефектоскопом в общем случае называют прибор, предназначенный для обнаружения и измерения дефектов. В этом смысле прибор Комплекс 2.05 не является дефектоскопом по утверждению разработчиков, его следует отнести к новому классу средств технической диагностики. Не всякий дефект в виде разрыва сплошности или инородного включения создает местную КМН или высокий градиент РГМН. Если в зоне контроля этим прибором имеется дефект, не создающий возмущение поля напряжений и не являющийся концентратором напряжений, то данный дефект на картограммах РГМН и КМН не будет обозначен. Наличие таких дефектов не препятствует безопасной эксплуатации металлоконструкции. В то же время любой существенный концентратор напряжений в виде дефекта даже весьма малых размеров или дефекта, вообще не имеющего нарушения (разрыва) сплошности среды и не обнаруживаемого традиционными методами дефектоскопии, может быть выявлен на карте РГМН и КМН. К ним могут быть отнесены, например, такие опасные дефекты, как тонкие трещины, зарождающаяся межкристаллитная коррозия и др.  [c.128]

В Краснодарском крае с 1987 г. ведутся работы по изучению ГГД поля с целью прогноза крупных землетрясений. В результате этой работы в 1994 г. бьши даны первые удачные прогнозы по землетрясениям средней интенсивности (М = 4,2-5,5). С этого года в крае прогнозировались практически все значимые землетрясения (М > 4,5). Успех прогноза зависит от плотности наблюдательной сети, возможности контроля активных разломных зон и оперативности получения и обработки информации. На текущий период в крае имеется 7 постов федеральной сети и 4 поста — краевой. К концу 2001 г. будет введено еще 4 поста краевой сети. Всего к концу 2001 г. в Краснодарском крае будет 15 наблюдательных постов. Все посты будут иметь телеметрическую связь и автоматизированный прием информации. Многолетний опыт показывает, что посредством изучения ГГД поля, можно в реальном времени видеть геодинамическое состояние территории, наблюдать формирование зон сжатия-растяжения, будущих очагов землетрясений, разнона-правленность движений литосферных блоков и возникновение между ними по разломной зоне критических полей напряжений, вызывающих вдоль них подвижки. Резкие изменения ГГД поля часто являются спусковым механизмом в активизации оползневых процессов [5]. Поэтому в системе безопасности эксплуатации линейных объектов методика изучения ГГД поля должна стать каркасной технологией ведения геодинамического мониторинга [6].  [c.37]

Весьма заманчивой возможностью для решения проблемы радиационной безопасности при космических полетах является создание так называемой активной защиты, использующей для отклонения заряженных частиц магнитные и электрические поля [30]. Вес такой защиты, как показывают оценки, в ряде случаев может быть сравнимым или меньще веса пассивной защиты. Важно также, что по мере совершенствования конструкционных и сверхпроводящих материалов, криогенной техники и техники сверхвысоких напряжений вес активной защиты будет снижаться [30].  [c.292]

На рис. 1-10 приводятся зависимости UrlU=f r) для заземлителей длиной /=10 40- -80 м. Из рассмотрения кривых следует, что чем длиннее заземлитель, тем более полого спадает его поле. Более пологая кривая распределения потенциала по поверхности земли ведет к снижению напряжения прикосновения Unp и шага Um, что существенно для безопасности при растекании тока промышленной частоты. Распространение поля от длин-24  [c.24]

При магнитном контроле целесообразно применение портативных самарий-кобаль-товых намагничивающих устройств, экологически безопасных суспензий. Намагничивающие устройства на основе высококоэрцитивных постоянных магнитов обеспечивают контроль во всех пространственных положениях, с плавной регулировкой напряженности магнитного поля. Для экологически чистого и быстрого магнитного контроля эффективно применение магнитных линз, аппликаторов, малогабаритных ультрафиолетовых излучателей и др.  [c.480]

Электробезопасность. Электрический ток вызывает сильные ожоги кожи, причем, степень поражения зависит от силы тока, продолжительности его воздействия, частоты и от состояния организма. Напряжение в сети 36 В относительно безопасно. Опасность поражения током увеличивается, если у рабочего мокрая одежда, влажные руки, если он стоит на мокром полу и пользуется инструментом (электродрелью, электробормашинкой и др.) напряжением 127—220 В. Во избежание поражения электрическим током необходимо следить за тем, чтобы изоляция на электрических проводах была исправна. При обнаружении неисправности в электрической части станка или машины необходимо сообщить об этом электрику и не приступать к работе до исправления повреждений.  [c.14]

Точки, расположенные внутри поля ОАБВСО, изображают безопасные циклы напряжений, не приводящие к усталостному разрушению. Точки же, расположенные вне этого поля, изображают циклы, приводящие к разрушению при числе нагружений, меньшем или равном базовому числу.  [c.402]

Травматизмы глаз чаще получают точильщики, обдирщики, шлифовщики, клепальщики, обрубщики, токари, слесари. Глаза могут быть поражены осколками мелких частиц от шлифовальных кругов, мелкой стружкой, окалиной металла и т. п. Поражение электрическим током вызывает сильные ожоги кожи, причем степень поражения зависит от силы тока, продолжительности его воздействия, частоты и от состояния организма. Ток свыше 0,1 а (100 л а) для человека смертелен. Напряжение 35—40 в относительно безопасно. Опасность поражения током увеличивается тогда, когда у рабочего мокрая одежда, влажные руки, если он стоит на мокром полу, если рабочий нездоров.  [c.33]

Для светильников, подвещиваемых на доступной высоте (2,5 м от пола и ниже), в сравнительно безопасных помещениях используется напряжение 127 или 220 В, а в помещениях с повыщенной опасностью и особо опасных — не выше 36 В. Для ручных переносных ламп в помещениях с повышенной опасностью допускается напряжение до 36 В, а в помещениях особо опасных и замкнутых металлических конструкциях — не выше 12 В.  [c.235]

Так, например, к рабочим средствам измерений, подлежащим обязательной государственной поверке, относятся весоизмерительные приборы, расходомеры, счетчики электроэнергии, газа, нефтепродуктов и воды, топливо- и маслораздаточные колонки и ряд других приборов, применяемых для учета и в торговле шумомеры дозиметры рентгенометры, сфигманометры и тонометры, наборы пробных очковых стекол, медицинские термометры и другие приборы, служащие для охраны здоровья трудящихся радиометры, измерители напряженности поля СВЧ, газоанализаторы, скоростемеры и другие измерительные приборы, обеспечивающие безопасность работ, и т. п. Все остальные средства измерений подлежат обязательной ведомственной поверке.  [c.204]

Техника безопасности при электроокрашивании. Специфической особенностью окрашивания изделий в электрическом поле является применение для нанесения лакокрасочных материалов электрического тока высокого напряжения, поэтому при эксплуатации электроустановок должны строго соблюдаться правила технической эксплуатации электроустановок потребителей и правила по технике безопасности при эксплуатации электроустановок потребителей.  [c.151]

Необходима прозвоночная лампа на напряжение, соответствующее напряжению цепи управления (50 или 110 В). Патрон лампы пластмассовый, ее провода длиной по 2 м должны быть снабжены щупами с изолированными ручками. Для постановки временных перемычек на рейках выводов и зажимах аппаратов и приборов удобно поль зоваться проводом площадью сечения 2,5 мм с зажимами типа Крокодил на концах. Таких перемычек нужно не менее двух на электропоезд. Следует также иметь н а электропоезде запас стеклянной и наждачной бумагя. Запасные предохранители хранят в каждом шкафу с электрооборудованием. При отыскании и устранении неисправностей необходимо строго соблюдать требования лравил техники безопасности.  [c.158]

Специфической особенностью окраски изделий в электрическом поле является применение для нанесения лакокрасочных материалов электрического тока высокого напряжения. Поэтому наряду с обеспечением требований пожаробезопасности, техники безопас-носги и санитарно-гигиенических требований, общих для всех методов окраски распылением, при эксплуатации электроокрасочных установок должны строго выполняться действующие Правила технической эксплуатации электроустановок потребителей и правила по технике безопасности при эксплуатации электроустановок потребителей , утвержденные с добавлениями Госииспекцией по промышленной энергетике и энергонадзору в 1961 г.  [c.132]

Электросварочные работы необходимо выполнять в кабинах, каркас которых изготавливают из труб или уголков, а стенки из тонкой листовой стали или брезента, пропитанного огнезащитным составом. Чтобы улучшить вентиляцию, стенкн не доводят до пола на 100— ЪОмм. Кабина должна хорошо освещаться, так как сварщику в процессе работы часто приходится проверять качество и правильность наплавки. Все оборудование и приспособления для электросварочных работ должны удовлетворять требованиям действующих правил безопасности и правил устройства электротехнических сооружений сильных токов низкого и высокого напряжения. Корпус сварочной машины или трансформатора, а также Свариваемая деталь должны быть надежно заземлены. Рабочий провод, подводящий ток от сварочной машины или трансформатора к электрододержателю, надежно изолируется и защищается от механических повреждений. Помимо щитков и шлемов с защитными стеклами, предохраняющими глаза от лучей электрической дуги, сварщики должны иметь очки для защиты глаз при очистке сварочных швов от шлака или окалины. Лица, работающие внутри сварочной кабины, должны иметь спецодежду и предохранительные приспособления такие же, как и электросварщики.  [c.182]

Особое значение при окрашивании в электрическом поле приобретают вопросы противопожарной безопасности. В условиях применения высокого напряжения и коронного разряда при нарушении электрического поля неизбежно возникает искровый разряд. При наличии в камере для окрашивания высокой концентрации паров растворителей искровой разряд может привести к их воспламенению.  [c.166]

Специфической особенностью нанесения полимерных материалов в электрическом поле является применение электрического тока высокого напряжения. При эсплуатации стационарных установок для нанесения материалов в электрическом поле помимо обеспечения общих требований по технике безопасности, пожароопасности и санитарно-технических требований необходимо все основное оборудование установок, находящееся под высоким напряжением (ИВН, шинопроводы, кабель, распыляющие устройства и др.), ограждать заземленными сетками, не допускающими свободный доступ к оборудованию. Обслуживающий персонал помимо обычных защитных средств обеспечивается изолирующими инструментами.  [c.236]

Применяемый в сигнализаторах отечественного и зарубежного производства принцип определения опасного расстояния до токонесущих проводов ЛЭП по уровню напряженности ее электрического поля не может обеспечить однозначную и надежную сигнализацию при опасном приближении конструктивных элементов стреловых механизмов к проводам ЛЭП и, следовательно, не позволяет гарантировать создания безопасных условий работы данных механизмов. Это обусловлено тем, что напряженность электрического поля, создаваемого ЛЭП, не определяется только напряжением ЛЭП и расстоянием до проводов, а зависит также от погодных условий, высоты подвеса проводов, их диаметра, расстояния между ними и конфигурации их пространственного размещения, наличия расположенных вблизи различных объектов (включая опоры ЛЭП), влияния конфигурации конструктивных элементов самого стрелового механизма и транспортируемого груза. Указанная неоднозначность особенно опасна при проведении работ в районе низковольтных ЛЭП (220/380 В), где допускаемое правилами безопасной работы расстояние между конструктивными элементами стрелового механизма и проводами ЛЭП составляет всего 1,5 м. Кроме того, этот принцип не обладает избирательностью по отношению к непосредственному источнику опасности, так как не позволяет различать близко расположенную низковольтную ЛЭП и далеко расположенную высоковольтную ЛЭП, не представляющую реальной опасности. Поэтому приборы типа УАС и УСОМ-ЭЛЕК-ТРОСТОП могут реагировать на более мошиую ЛЭП, находящуюся на безопасном расстоянии, или срабатывать в зоне нескольких ЛЭП. В то же время они неработоспособны при наличии на кране бортовой питающей сети переменного тока 50 Гц и не реагируют на ЛЭП постоянного тока.  [c.278]

За последнее время большой интерес приобрел новый метод окраски в электрическом поле с применением ручных электростатических распылителей. Установки ручной электроокраски состоят из электростатического устройства, краскоподающей аппаратуры и распылителя. Постоянный электрический ток подается к распылителю напряжением до 90 тыс. в и силой тока 0,2 ма. Так как сила тока при данном напряжении в сто раз меньше опасной для человеческой жизни, работа распылителем вполне безопасна. Подобные установки выпускает предприятие Комплекс (Венгрия) [46], фирма Самес (Франция  [c.150]

Нормаль ЭНИМСа Н06-2 предусматривает рекомендации для цветов окраски наружных и внутренних частей станков, предназначенных для различных условий эксплуатации, в том числе станков, предназначенных для работы в районах с тропическим климатом. Для окраски наружных поверхностей станка рекомендуются цвета сложных оттенков серый, светло-серый, фисташковый и зелено-голубой в кремовый окрашиваются отдельные детали и узлы при двухцветной окраске. Для окраски внутренних полостей станков применяются кремовый и серебристый цвет. В красный цвет окрашиваются устройства для останова процесса или движения кнопки стоп и рукоятки выключения, фон для быстро-перемещающихся деталей и механизмов. В желтый цвет окрашиваются кромки ограждающих устройств и особо опасные подвижные элементы. Белый и кремовый цвета используются в качестве фона для черных делительных шкал, поясняющих надписей, для внутренних поверхностей электрошкафов и пультов. В черный цвет окрашиваются заземляющие шины. На трубопроводах станков наносится цветное кольцо светло-коричневое — для смазочно-охлаждаЪщих жидкостей голубое — для сжатого воздуха красное — для электроприводов под рабочим напряжением. Для безопасного обслуживания станков предусматриваются ограждения движущихся деталей, главным образом патронов, ходового винта, валика и др., защитные устройства, предотвращающие попадание на рабочего и на пол стружки, охлаждающей жидкости и смазки станки должны иметь индивидуальный привод. Рукоятки, маховики со спицами и ручками, быстро вращающиеся при ускоренных перемещениях, должны отключаться во время этих перемещений рычаги, управляющие несовместимыми движениями, должны снабжаться устройствами, исключающими возможность их одновременного включения.  [c.235]

Ламповые генераторы или генераторы колебаний, преобразующие электрическую энергию в высокочастотное поле, являются основной составной частью всех видов оборудования для диэлектрического нагрева. Генераторы, применяемые в оборудовании для высокочастотной сварки пластмасс, обеспечивают выходную мощность от 1 до 50 кет и обычно работают с частотами от 2 до 100 мггц при напряжении от 4000 до 12 ООО в. Рабочее напряжение должно быть по возможности большим, однако ниже той точки, при которой полное расплавление и растекание материала будет происходить очень быстро. Обычно вначале напряжение в оборудовании для высокочастотной сварки устанавливается до такой величины, при которой происходит расплавление материала, а затем напряжение понижается до значения, обеспечивающего безопасный рабочий режим сварных операций. Частота колебаний в оборудовании должна быть установлена не выше 200 мггц, поскольку работа с большими частотами колебаний переменного тока связана с рядом дополнительных трудностей в отношении выработки электроэнергии и применения оборудования. Установки для высокочастотной сварки, потребляющие мощность от 4 до б кет, выпускаются в качестве стандартного оборудования для нестандартного назначения может быть изготовлено и поставлено оборудование специальной конструкции.  [c.126]


Электрическое напряжение. Вольтметр — урок. Физика, 8 класс.

Пробовали ли вы когда-нибудь надувать воздушные шарики на время? Один надувает быстро, а другой за это же время надувает гораздо меньше. Без сомнения, первый совершает большую работу, чем второй.

 

 

С источниками напряжения происходит точно так же. Чтобы обеспечить движение частиц в проводнике, надо совершить работу. И эту работу совершает источник. Работу источника характеризует напряжение. Чем оно больше, тем большую работу совершает источник, тем ярче будет гореть лампочка в цепи (при других одинаковых условиях).

 

Напряжение равно отношению работы электрического поля по перемещению заряда
к величине перемещаемого заряда на участке цепи.

U=Aq, где \(U\) — напряжение, \(A\) — работа электрического поля, \(q\) — заряд.

 

Обрати внимание!

Единица измерения напряжения в системе СИ — [\(U\)] = \(1\) B (вольт).

\(1\) вольт равен электрическому напряжению на участке цепи, где при протекании заряда, равного \(1\) Кл, совершается работа, равная \(1\) Дж: \(1\) В \(= 1\) Дж/1 Кл.

Все видели надпись на домашних бытовых приборах «\(220\) В». Она означает, что на участке цепи совершается работа \(220\) Дж по перемещению заряда \(1\) Кл.

 

Кроме вольта, применяют дольные и кратные ему единицы — милливольт и киловольт.

\(1\) мВ \(= 0,001\) В, \(1\) кВ \(= 1000\) В или \(1\) В \(= 1000\) мВ, \(1\) В \(= 0,001\) кВ.

Для измерения напряжения используют прибор, который называется вольтметр.

Обозначаются все вольтметры латинской буквой \(V\), которая наносится на циферблат приборов и используется в схематическом изображении прибора.

 

 

В школьных условиях используются вольтметры, изображённые на рисунке:

 

 

 

Основными элементами вольтметра являются корпус, шкала, стрелка и клеммы. Клеммы обычно подписаны плюсом или минусом и для наглядности выделены разными цветами: красный — плюс, черный (синий) — минус. Сделано это с той целью, чтобы заведомо правильно подключать клеммы прибора к соответствующим проводам, подключённым к источнику.

 

Обрати внимание!

В отличие от амперметра, который включается в разрыв цепи последовательно, вольтметр включается в цепь параллельно.

 

Включая вольтметр в цепь постоянного тока, необходимо соблюдать полярность.

 

Сборку электрической цепи лучше начинать со всех элементов, кроме вольтметра, а его уже подключать в самом конце.

Вольтметры делятся на приборы постоянного тока и переменного тока.

Если прибор предназначен для цепей переменного тока, то на циферблате принято изображать волнистую линию. Если прибор предназначен для цепей постоянного тока, то линия будет прямой.

 

Вольтметр постоянного тока

Вольтметр переменного тока

 

Можно обратить внимание на клеммы прибора. Если указана полярность («\(+\)» и «\(-\)»), то это прибор для измерения постоянного напряжения.


Иногда используют буквы \(AC/DC\). В переводе с английского \(AC\) (alternating current) — переменный ток, а \(DC\) (direct current) — постоянный ток.
В цепь переменного тока включается вольтметр для измерения переменного тока. Он полярности не имеет.

 

 

Обрати внимание!

Для измерения напряжения можно использовать и мультиметр.

Перед измерением необходимо прочитать инструкцию, чтобы правильно подключить прибор.

 

 

Следует помнить, что высокое напряжение опасно.

Что будет с человеком, который окажется рядом с упавшим оголённым кабелем, находящимся под высоким напряжением?

Так как земля является проводником электрического тока, вокруг упавшего оголённого кабеля, находящегося под напряжением, может возникнуть опасное для человека шаговое напряжение.

 

При попадании под шаговое напряжение даже небольшого значения возникают непроизвольные судорожные сокращения мышц ног. Обычно человеку удаётся в такой ситуации своевременно выйти из опасной зоны.

 

Обрати внимание!

Однако нельзя выбегать оттуда огромными шагами, шаговое напряжение при этом только увеличится! Выходить надо обязательно быстро, но очень мелкими шагами или скачками на одной ноге!

Существует много знаков, предупреждающих о высоком напряжении. Вот некоторые из них.

 

   

 

Безопасным напряжением для человека считается напряжение \(42\) В в нормальных условиях и \(12\) В в условиях с повышенной опасностью (сырость, высокая температура, металлические полы и др.).

Источники:

Пёрышкин А.В. Физика, 8 класс// ДРОФА, 2013.

http://class-fizika.narod.ru/8_29.htm
http://interneturok.ru/ru/school/physics/8-klass/belektricheskie-yavleniyab/elektricheskoe-napryazhenie

http://kamenskih3.narod.ru/untitled74.htm

Вопросы электробезопасности при тушении пожара

В рубрику «Пожарная безопасность» | К списку рубрик  |  К списку авторов  |  К списку публикаций

Вопросы электробезопасности при тушении пожара

Говоря о работах в области пожарной безопасности, невозможно обойти вниманием тему электропитания противопожарного оборудования. Чтобы обеспечить эффективность использования противопожарных средств и систем, их необходимо постоянно поддерживать в состоянии работоспособности, то есть под напряжением. Для этого используется электропитание от источников тока с малым напряжением. Но величина малого напряжения варьируется в достаточно больших границах и далеко не все величины напряжения могут считаться безопасными для человека

Действующие отечественные нормы и практика

В соответствии с требованиями и нормами пожарной безопасности (НПБ), строительными нормами и правилами (СНиП), правилами устройства электроустановок (ПУЭ) и государственными стандартами России (ГОСТ Р) электроснабжение технических средств пожарной сигнализации, пожарного оповещения, дымоудаления и пожаротушения осуществляется от источника электроэнергии 1 -й категории. Электроснабжение этих средств не должно отключаться при любых неблагоприятных условиях. В критических ситуациях предусматривается переход на резервное электропитание, в том числе автономное. Включение в цепи электропитания этих средств устройств защитного отключения (УЗО) не допускается. Для поддержания работоспособности противопожарных технических средств они всегда находятся под напряжением, даже в случае аварийного отключения основных источников электроснабжения. Подобные требования логичны, разумны и направлены на противодействие возникновению и распространению пожара и спасение жизни и здоровья людей. В условиях возникновения пожара технические средства пожарной сигнализации (охранной сигнализации), пожарного оповещения, дымоудаления, пожаротушения должны продолжать исправно действовать.

В целях уменьшения опасности поражения электрическим током значительная часть указанных технических средств, в соответствии с упомянутыми отечественными нормами, рассчитана на электропитание от источников тока с малым электрическим напряжением. Это «малое напряжение», указанное прямо или косвенно в различных нормах, ведомственных руководящих документах и рекомендациях, колеблется в широких пределах: от 9 до 72 В (табл. 1).

Поскольку для профессиональных разработчиков аппаратуры и проектировщиков зданий и сооружений отечественные нормы являются руководством к действию, то практически на всех действующих и строящихся объектах, снабженных средствами пожарной безопасности, имеются электрические цепи, постоянно находящиеся под напряжением от 9 до 72 В.

Не является ли напряжение, регламентируемое государственными и ведомственными нормами и закрепленное в перечнях технических средств, разрешенных к применению, опасным, особенно в условиях тушения пожара? Какое напряжение является действительно безопасным? Попытаемся разобраться в этом вопросе.

Последствия воздействия электрического тока на человека

Степень опасности воздействия электрического тока на человека зависит от двух факторов: силы тока и времени протекания тока через тело человека. Градации степени опасности воздействия электрического тока на человека приведены в документах Международной электротехнической комиссии (МЭК) [11]. Эти градации показаны на рис. в виде графиков. Переменный (а) и постоянный (б) токи воздействуют на человека по-разному. На графиках по степени опасности выделены четыре основные зоны. Физиологические последствия от воздействия электрического тока на человека в этих зонах описаны в табл. 2.

Максимально допустимое время протекания электрического тока через тело человека в зависимости от напряжения прикосновения и полного сопротивления цепи, полученное путем расчетов, отраженных в графиках на рис., дано в табл. 3 и 4.

Нормы на предельно допустимые значения напряжений прикосновения и токов, регламентируемые в нашей стране ГОСТ 12.1.038-82 [12] по электробезопасности, значительно более жесткие, чем в МЭК. Мало того, в условиях высоких температур (> 25 °С) и влажности (> 75%) допустимые напряжения и токи в соответствии с ГОСТом должны быть уменьшены в 3 раза. Следует заметить, что практически во всех нормативных документах, приведенных в табл. 1, даже в таком фундаментальном документе (с точки зрения глубины системной проработки), как НПБ 88-2001 [10], не учтены требования действующего стандарта по электробезопсности ГОСТ 12.1.038-82.

Условия тушения пожара

Самым распространенным огнетушащим веществом в нашей стране является вода, хорошо проводящая электрический ток. Применение пенообразователей для повышения огнетушащих свойств смеси приводит к повышению электропроводности воды.

При тушении пожара человек находится в условиях высокой температуры и влажности (из-за обилия воды), что способствует повышению электропроводности его кожи, одежды, окружающих предметов и увеличению силы тока, который может проходить через тело человека. Здесь уместно напомнить, что электроснабжение средств пожарной безопасности не прекращается во время пожара даже при отключении основных источников электроснабжения. И контакт с цепями, находящимися под напряжением от 9 до 72 В, возможен в любой момент.

В строительных нормах МЭК, относящихся к устройству электроустановок [13, 14], даны общие рекомендации по электробезопасности. Особенно интересен документ, регламентирующий устройство электроустановок в электроопасных зонах с повышенной влажностью и большими объемами воды: в ванных комнатах, душевых, бассейнах и т.п. [15]. Эти условия близки к условиям тушения пожара в обычных («сухих») помещениях. В соответствии с этим документом безопасным для человека считается напряжение 12 В и менее. Следовательно, напряжение выше 12 В следует признать потенциально опасным в условиях тушения пожара. Его не рекомендуется использовать для электропитания неотключаемых систем и устройств.

Предварительные выводы

В результате анализа перечисленных нормативных документов можно констатировать следующее:

  • Большая часть нормативных документов Госстроя, МВД и Минтопэнерго России, относящихся к построению систем пожарной, охранной, охранно-пожарной сигнализации, оповещения, дымоудаления, пожаротушения содержит противоречивые сведения, связанные с обеспечением электробезопасности. В частности, малое («безопасное») напряжение, применяемое в технических средствах пожарной безопасности, указывается в пределах от 9 до 72 В. Эти документы в части электробезопасности противоречат международным нормам МЭК и требованиям отечественного стандарта ГОСТ 12.1.038-82.
  • Во время тушения пожара традиционными средствами создаются условия, при которых опасным с точки зрения поражения электрическим током оказывается напряжение выше 12 В, которое всегда присутствует в цепях, укрепленных с точки зрения пожарной охранной безопасности объектов. При попадании человека под такое напряжение еще не наблюдается биологическое поражение тканей его тела, но происходят физиологические изменения, которые приводят к затруднению дыхания, желудочковой фибриляции и могут привести к остановке сердца.
  • Здания и сооружения, построенные в соответствии с действующими отечественными ведомственными нормами (СНиП, НПБ, ПУЭ и др.), оказываются опасными для людей в смысле поражения электрическим током при тушении в них пожара.
  • В электрических цепях систем пожарной, охранной, охранно-пожарной сигнализации, оповещения, пожаротушения и других систем, в которых не предусматривается аварийное или защитное отключение электроснабжения, следует использовать напряжение не более 12 В, которое не приводит к поражению людей электрическим током.

Что делать

Описанную ситуацию, которая десятилетиями складывалась при разработке ведомственных нормативных документов без учета действующих отечественных и международных стандартов, моментально изменить нельзя. Слишком большим весом обладают СниПы, НПБ, ПУЭ для многомиллионной армии строителей, проектировщиков и владельцев недвижимости. Но менять ее необходимо. Предлагается следующее:

  • Заинтересованным ведомствам и организациям (МЧС, МВД, Госстрой, Минтопэнерго, Госстандарт России, НИИПО, НИЦ «Охрана» МВД и др.) необходимо убедиться в достоверности изложенных фактов и создать межведомственную рабочую группу по пересмотру и гармонизации перечисленных и иных нормативных документов с целью обеспечения электробезопасности зданий и сооружений, в том числе при тушении пожара.
  • При пересмотре норм основываться на указанном ГОСТе и международных стандартах МЭК.
  • По мере разработки новых норм заблаговременно информировать через средства массовой информации техническую и деловую общественность, для того чтобы предприятия и организации, занятые разработкой и производством технических средств, имели резерв времени для перестройки производства и выпуска новой безопасной продукции. Срок введения новых норм с той же целью назначать не ранее 6 месяцев после их утверждения и опубликования.

Список источников

  1. ГОСТ 12.1. 009-76. Система стандартов безопасности труда. Электробезопасность. Термины и определения.
  2. Правила устройства электроустановок/Минэнерго СССР. 6-е изд., М.,: Энергоатомиздат, 1985 г.
  3. Правила устройства электроустановок. Раздел 6. Электрическое освещение. Раздел 7. Электрооборудование специальных установок. Глава 7.1. Электроустановки жилых, общественных административных и бытовых зданий. Глава 7.2. Электроустановки зрелищных предприятий, клубных учреждений и спортивных сооружений. 7-е издание. М., «Изд. НЦ ЭНАС», 1999 г.
  4. РД 78.145-93. Системы и комплексы охранной, пожарной и охранно-пожарной сигнализации. Правила производства и приемки работ/ГУВО МВД России. М., 2000
  5. Пособие к руководящему документу Системы и комплексы охранной, пожарной и охранно-пожарной сигнализации. Правила производства и приемки работ РД 78.145-93./ГУ-ВО МВД России. М., 1995 г.
  6. СНиП 2.04.09-84. Пожарная автоматика зданий и сооружений.
  7. Р 78.36.007-99. Выбор и применение средств охранно-пожарной сигнализации и средств технической укрепленности для оборудования объектов — Рекомендации./ГУВО МВД России. М., 1998 г.
  8. РД 78.143-92. Системы и комплексы охранной сигнализации. Элементы технической укрепленности объектов. Нормы проектирования./ГУВО МВД России. М., 2000 г.
  9. П 78.36.002-2000. Перечень средств вневедомственной охраны, разрешенных к применению в 2000 г./ГУВО МВД России. М., 2000 г.
  10. НПБ 88-2001. Нормы пожарной безопасности. Установки пожаротушения и сигнализации. Нормы и правила проектирования./ГПС МВД России. М., 2002 г.
  11. IEC/TR 60479-2 (1994-09) Effects of current on human beings and livestock. Part 2: Special aspects. — Chapter 4: Effects of alternating current with frequencies above 100 Hz; Chapter 5: Effects of special waveform of current; Chapter 6: Effect unidirectional signal impulse currents of short duration. — 44 pp.
  12. ГОСТ 12.1.038-82. Система стандартов безопасности труда. Электробезопасность. Предельно допустимые значения напряжения прикосновения и токов.
  13. IEC 60364-3 (1993-03) Electrical installation of buildings. Part 3: Assessment to general characteristics. — 55 pp. + 600364-3 am 1 (1994-02), + 600364-3 am 2 (1995-12).
  14. IEC 60364-4-41 (1992-10) Electrical installation of buildings. Part 4: Protection for safety — Chapter 41. Protection against electric shock. — 49 pp. + 600364-4-41 am1 (1966-02) + 600364-4-41 am2 (1999) (TN, TT, IT earthing).
  15. IEC 60364-7-701 Ed. 1.0 Electrical installations of buildings. Part 7: Requirements for special installations or locations. Section 701: Electrical installations in bathrooms.

В.И. Щербина
Технический директор ООО «БИС Инжинеринг», к.т.н.

Опубликовано: Каталог «Пожарная безопасность»-2003
Посещений: 15510

В рубрику «Пожарная безопасность» | К списку рубрик  |  К списку авторов  |  К списку публикаций

9. Электрический ток. Допустимые значения токов и напряжений

Наша современная жизнь полна разнообразием бытовых приборов и устройств, которые существенно облегчают нам быт, делают его все более комфортным, но одновременно появляется целый комплекс опасных, вредных факторов: электромагнитные поля различных частот, повышенный уровень радиации, шумы, вибрации, опасности механического травмирования, наличие токсичных веществ, а так же самое главное – электрический ток.

Электрическим током называется упорядоченное движение электрических частиц. Для вашей же безопасности необходимо знать действие электрического тока на организм человека, меры защиты от поражения током, оказание помощи пострадавшему от воздействия электротока человеку.

Воздействие на организм человека электрического тока

На человека электрический ток оказывает биологическое, термическое, электролитическое действия.

Термическое: нагревание тканей при протекании по ним электрического тока.

Электролитическое: разложение крови и других жидкостей организма.

Биологическое: возбуждение живых тканей организма, сопровождается судорогами, спазмом мышц, сердечной деятельностью, остановкой дыхания.

Когда на человека действует электрический ток, возникают телесные электротравмы: ожоги, электрические знаки, металлизация кожи, механические повреждения, ослепление светом электрической дуги, или может произойти электрический удар – это общее поражение организма, которое может сопровождаться судорогами, потерей сознания, остановкой дыхания и сердца, и даже клинической смертью.

Электрические знаки – это пятна серого и бледно-желто цвета, ушибы, царапины на коже человека, которые подвергались действию тока. Сила знака соответствует силе токоведущей части, которой коснулся человек. В большинстве случаев лечение электрических знаков заканчивается благополучно, а пораженное место полностью восстанавливается.

Механические повреждения возникают под действием электрического тока, когда непроизвольно судорожно сокращаются мышцы. Механические повреждения (переломы костей, разрывы кровеносных сосудов, кожи) это повреждения, которые требуют долгого лечения.

Удар электрическим током. Время от времени бывают случаи, когда дети из любопытства засовывают пальцы в электрическую розетку или начинают ковырять в ней гвоздем, проволокой или другими металлическими предметами. Чаще всего это бывает с детьми до трех лет. Бывают случаи, когда дети получают удар электрическим током от упавших на землю и находящихся под напряжением проводов. При воздействии электрического тока на организм может возникнуть непроизвольное судорожное сокращение мышц, мешающее ребенку оторваться от источника тока. В месте соприкосновения с током возникает электроожог. В тяжелом случае появляется расстройство дыхания и сердечной деятельности. Первое, что нужно сделать, – освободить ребенка от действия электрического тока. Самое безопасное – быстро вывернуть пробки, если несчастный случай произошел в доме. Если по каким-либо причинам это сделать невозможно, то необходимо бросить себе под ноги резиновый коврик, доску или толстую ткань либо надеть на ноги резиновые сапоги или галоши; можно надеть на руки хозяйственные резиновые перчатки. Пострадавшего оттащить от провода, схватившись одной рукой за одежду. Можно также попытаться отодвинуть самого пострадавшего от источника тока либо отстранить от него источник. Сделать это нужно одной рукой, чтобы даже при получении удара ток не прошел через все тело того, кто оказывает помощь. Пострадавшего необходимо уложить, тепло укрыть, освободить от стесняющей одежды, при возможности дать теплое питье. На обожженный электротоком участок тела следует наложить стерильную повязку из бинта или чистой ткани, предварительно смочив ее в спирте или водке. Если ребенок потерял сознание, ему дают понюхать нашатырный спирт и брызгают в лицо холодной водой. Если ребенок лежит без сознания и у него отсутствует дыхание, но есть пульс, необходимо немедленно делать ему искусственное дыхание методом «рот в рот». Для этого голову ребенка запрокидывают назад и, зажимая ему ноздри, вдувают в рот воздух порциями, приложив свои губы к губам ребенка.

Электрический ожог разных степеней – результат коротких замыканий в электрических установках и нахождение тела (рук) в среде светового и теплового влияния электрической дуги; ожоги III и IV степени с тяжелым исходом – при соприкосновении человека с частями, по которым проходит ток напряжением свыше 1000 В.

Металлизация кожи это мельчайшие частицы металла проникают в верхние слои кожи, расплавившегося под действием электрической дуги или растворенного в электролитах электролизных ванн. В пораженном месте кожа становится жесткой, шероховатой и приобретает ту окраску какая у металла (например, зеленую – от соприкосновения с медью). Работа, связанная с вероятностью возникновения электрической дуги, следует делать в очках, а одежда работника должна быть застегнута на все пуговицы.

Сила тока ,mA

Переменный ток

Постоянный ток

0,6 -1,5

Ощущение протекания тока Пальцы рук дрожат (легко)

Не ощущается

2 – 3

Пальцы рук дрожат (сильно)

Не ощущается

10-15

Судороги в руках

Зуд. Ощущение нагрева

20 – 25

Руки парализуются немедленно, оторвать их от электродов не возможно, очень сильные боли. Дыхание затруднено

Еще больше усиливается нагревание, незначительное сокращение мышц рук

50 – 80

Паралич дыхания. Начинаются трепетать желудочки сердца

Сильное ощущение нагревания. Сокращение мышц рук. Судороги. Затруднение дыхания.

100

Фибрилляция сердца

Паралич дыхания

Электроофтальмия – ультрафиолетовый луч (источником которых, является вольтова дуга, она поражает глаз). В результате электроофтальмии наступает воспалительный процесс, и если приняты необходимые меры лечения, то боль проходит.

В зависимости от величины тока, его напряжения, частоты, продолжительности воздействия, пути тока и общего состояния человека зависит исход действия электрического тока на организм человека. установлено, что ток силой более 0,05 А может смертельно травмировать человека в течение 0,1 с. Самое большое число поражений от электрического тока (около 85%) приходится на установки напряжением до 1000 В. Для человеческого организма опасны переменный и постоянный ток. Наиболее опасен переменный ток, имеющий частоту 20-100 Гц; а частота 400 Гц не так опасна. Практически безопасным для человека в сырых помещениях можно считать напряжение до 12 В, в сухих помещениях – до 36 В. Вероятность поражения человека электрическим током зависит от климатических условий в помещении (температуры, влажности), а также токопроводящей пыли, металлических конструкций, соединенных с землей, токопроводящего пола и т.д.

В соответствии с «Правилами устройства электроустановок потребителей» (ПУЭ) все помещения делят на три класса:

  • без повышенной опасности – нежаркие (до +35°С), сухие (до 60%), непыльные, с нетокопроводящим полом, не загроможденные оборудованием;

  • с повышенной опасностью – имеют, по крайней мере, один фактор повышенной опасности, т.е. жаркие или влажные (до 75%), пыльные, с токопроводящим полом и т.п.;

  • особо опасные – имеют два или более факторов повышенной опасности или, по крайней мере, один фактор особый опасности, т.е. особую сырость (до 100%) или наличие химически активной среды.

Возможные значения токов и напряжений соприкосновения в зависимости от времени срабатывания защиты указаны в ГОСТ 12.1.038-88. По этому документу для нормального (неаварийного) режима работы промышленного оборудования допустимые напряжения прикосновения не должны быть больше 2 В при частоте тока 50 Гц, 3 В при 400 Гц и 8 В для постоянного тока, но суммарная продолжительность воздействия не должна превышать 10мин в сутки. В нормальном режиме работы бытовой аппаратуры наличие напряжений прикосновения не допускается. В особо опасных (или с повышенной опасностью) помещениях подлежит заземлению все оборудование при напряжении питания свыше 42В переменного и ПО В постоянного тока. В нормальных помещениях все оборудование при напряжении 380 В и выше переменного и 440 В и выше постоянного тока. Все оборудование независимо от напряжения питания заземляется только во взрывоопасных помещениях.

С увеличением продолжительности воздействия электрического тока на человека возрастает угроза поражения. Через 30 сек. сопротивление тела человека протеканию тока падает примерно на 25%, через 90 сек. на 70%. Сопротивление организма человека электрическому току колеблется в широком диапазоне. Сухая, грубая мозолистая кожа, отсутствие усталости и нормальное состояние нервной системы повышает сопротивление человеческого организма. Нервные волокна и мускулы обладают наименьшим сопротивлением. За минимальное расчетное сопротивление человеческого организма принимается величина от 500 до 1000 Ом.

В тот момент, когда человек замыкает своим телом два фазных провода действующей установки, он попадает под полное линейное напряжение сети. При учете того, что расчетное сопротивление тела человека принимается 1000 Ом, то при двухфазном прикосновении к действующим частям установки, напряжение в которой 100 В, может оказаться смертельным, по причине того, что ток, проходящий через тело человека, достигает величины 0,1 А.

Если через тело человека проходит ток 0,06 А и более, происходит поражение электрическим током. Сопротивление человека электрическим током величина переменная. Она зависит от многих факторов, в том числе от психологического состояние и физического состояния человека. В пределах 20-100 кОм находится среднее значение сопротивления. Оно может снизиться до 1 кОм при особо неблагоприятных условиях. В этом случае окажется опасным для жизни человека напряжение 100 В и ниже.

Величина тока, проходящая через человеческое тело, зависит от его сопротивления. А сопротивление зависит в основном от состояния кожи человека. Сопротивления тела человека зависит и от частоты тока. За расчетную величину электрического сопротивления тела принято сопротивление, равное 1,0 кОм. При частотах тока 6-15 кГц оно бывает наименьшим.

Постоянный ток является менее опасным, чем переменный. Постоянный ток до 6 мА почти не ощутим. При токе 20 мА появляются судороги в мускулах предплечья. Переменный ток начинает ощущаться уже при 0,8 мА. Ток 15 мА вызывает сокращение мышц рук. Особенно опасным является прохождение тока через сердце.

Опасность поражения постоянным и переменным током изменяется с увеличением напряжения. При напряжении до 220 В более опасным является переменный ток, а при напряжении выше 500 В опасное постоянный ток. Чем больше протекает ток, тем меньше становится сопротивление человеческого тела. Может наступить смерть, если действие электрического тока не будет прервано. Если ток проходит от руки к ногам, то существенное значение имеет какая на человеке обувь, из какого она материала, какого она качества. На степень поражения значительное влияние оказывает также сопротивление в месте соприкосновения человека с землей. Электрический ток имеет тяжелые последствия, вплоть до остановки сердца и прекращения дыхания. Поэтому нужно уметь оказать первую помощь пострадавшему от поражения электрическим током.

Статическое электричество – это потенциальный запас электрической энергии, образующейся на оборудовании в результате трения, индукционного влияния сильных электрических разрядов. В помещениях с большим кол-вом пыли органического происхождения могут образоваться статические разряды, а также накапливаться на людях при пользовании бельем и одеждой из щелка, шерсти и искусственных волокон, при движении по токонепроводящему синтетическому покрытию пола, типа линолеума, кавролина и т.д.

Нормирование электростатического поля проводится в соответствии с ГОСТ 12.1.045-84 напряженность электрического поля на рабочих местах не должна превышать 60 кВ/м в течение часа. Время пребывания в электрическом поле при 20≤Е≤60 (кВ) рассчитывается по формуле t=(60/E)2, где Е – фактическое значение напряженности поля. Сопротивление заземляющих устройств для защиты от статического электричества не должно превышать 100 (Ом).

ССБТ. Электробезопасность. Предельно допустимые значения напряжений прикосновения и токов

МЕЖГОСУДАРСТВЕННЫЙ СТАНДАРТ

Система стандартов безопасности труда

ЭЛЕКТРОБЕЗОПАСНОСТЬ

Предельно допустимые значения напряжений прикосновения и токов

Occupational safety standards system. Electric safety.

Maximum permissible values of pick-up voltages and currents

ГОСТ
12.1.038-82*

Постановлением Государственного комитета СССР по стандартам от 30 июля 1982 г. № 2987 дата введения установлена

01.07.83

Ограничение срока действия снято по протоколу № 2-92 Межгосударственного Совета по стандартизации, метрологии и сертификации (ИУС 2-93)

 

Настоящий стандарт устанавливает предельно допустимые значения напряжений прикосновения и токов, протекающих через тело человека, предназначенные для проектирования способов и средств защиты людей, при взаимодействии их с электроустановками производственного и бытового назначения постоянного и переменного тока частотой 50 и 400 Гц.

Термины, используемые в стандарте, и их пояснения приведены в приложении.

(Измененная редакция, Изм. № 1).

1.1 Предельно допустимые значения напряжений прикосновения и токов установлены для путей тока от одной руки к другой и от руки к ногам.

(Измененная редакция, Изм. № 1).

1.2 Напряжения прикосновения и токи, протекающие через тело человека при нормальном (неаварийном) режиме электроустановки, не должны превышать значений, указанных в табл. 1.

 

Таблица 1

 

Род тока

U, В

I, мА

не более

Переменный, 50 Гц

2,0

0,3

Переменный, 400 Гц

3,0

0,4

Постоянный

8,0

1,0

 

Примечания:

1 Напряжения прикосновения и токи приведены при продолжительности воздействий не более 10 мин в сутки и установлены, исходя из реакции ощущения.

2 Напряжения прикосновения и токи для лиц, выполняющих работу в условиях высоких температур (выше 25°С) и влажности (относительная влажность более 75%), должны быть уменьшены в три раза.

 

1.3 Предельно допустимые значения напряжений прикосновения и токов при аварийном режиме производственных электроустановок напряжением до 1000 В с глухозаземленной или изолированной нейтралью и выше 1000 В с изолированной нейтралью не должны превышать значений, указанных в табл. 2.

 

Таблица 2

 

Род тока

Нормируемая величина

Предельно допустимые значения, не более, при продолжительности воздействия тока t, с

0,01-0,08

0,1

0,2

0,3

0,4

0,5

0,6

0,7

0,8

0,9

1,0

Св.1,0

Переменный 50 Гц

U, B

550

340

160

135

120

105

95

85

75

70

60

20

I, мА

650

400

190

160

140

125

105

90

75

65

50

6

Переменный 400 Гц

U, B

650

500

500

330

250

200

170

140

130

110

100

36

I, мА

 

 

 

 

 

 

 

 

 

 

 

8

Постоянный

U, B

650

500

400

350

300

250

240

230

220

210

200

40

I, мА

 

 

 

 

 

 

 

 

 

 

 

15

Выпрямленный двухполупериодный

Uампл, B

650

500

400

300

270

230

220

210

200

190

180

Iампл, мА

 

 

 

 

 

 

 

 

 

 

 

 

Выпрямленный однополупериодный

Uампл, B

650

500

400

300

250

200

190

180

170

160

150

Iампл, мА

 

 

 

 

 

 

 

 

 

 

 

 

 

Примечание — Предельно допустимые значения напряжений прикосновения и токов, протекающих через тело человека при продолжительности воздействия более 1 с, приведенные в табл. 2, соответствуют отпускающим (переменным) и неболевым (постоянным) токам.

 

1.4 Предельно допустимые значения напряжений прикосновения при аварийном режиме производственных электроустановок с частотой тока 50 Гц, напряжением выше 1000 В, с глухим заземлением нейтрали не должны превышать значений, указанных в табл. 3.

Таблица 3

 

Продолжительность воздействия t, с

Предельно допустимое значение напряжения прикосновения U, в

До 0,1

500

0,2

400

0,5

200

0,7

130

1,0

100

Св. 1,0 до 5,0

65

 

1.5 Предельно допустимые значения напряжений прикосновения и токов при аварийном режиме бытовых электроустановок напряжением до 1000 В и частотой 50 Гц не должны превышать значений, указанных в табл. 4.

 

Таблица 4

 

Продолжительность воздействия t, с

Нормируемая величина

Продолжительность воздействия t, c

Нормируемая величина

U, B

I, мА

U, B

I, мА

От 0,01 до 0,08

220

220

0,6

40

40

0,1

200

200

0,7

35

35

0,2

100

100

0,8

30

30

0,3

70

70

0,9

27

27

0,4

55

55

1,0

25

25

0,5

50

50

Св. 1,0

12

2

 

Примечание — Значения напряжений прикосновения и токов установлены для людей с массой тела от 15 кг.

1.3-1.5 (Измененная редакция, Изм. № 1).

1.6 Защиту человека от воздействия напряжений прикосновения и токов обеспечивают конструкция электроустановок, технические способы и средства защиты, организационные и технические мероприятия по ГОСТ 12.1.019-79.

2.1 Для контроля предельно допустимых значений напряжений прикосновения и токов измеряют напряжения и токи в местах, где может произойти замыкание электрической цепи через тело человека. Класс точности измерительных приборов не ниже 2,5.

2.2 При измерении токов и напряжений прикосновения сопротивление тела человека в электрической цепи при частоте 50 Гц должно моделироваться резистором сопротивления:

для табл. 1 — 6,7 кОм;

для табл. 2 при времени воздействия

до 0,5 с -0,85 кОм;

более 0,5 с — сопротивлением, имеющим зависимость от напряжения согласно чертежа;

для табл. 3 — 1 кОм;

для табл. 4 при времени воздействия

до 1 с — 1 кОм;

более 1 с — 6 кОм.

Отклонение от указанных значений допускается в пределах ±10%.

 

(Измененная редакция, Изм. № 1).

 

2.3 При измерении напряжений прикосновения и токов сопротивление растеканию тока с ног человека должно моделироваться с помощью квадратной металлической пластины размером 25´25 см, которая располагается на поверхности земли (пола) в местах возможного нахождения человека. Нагрузка на металлическую пластину должна создаваться массой не менее 50 кг.

2.4 При измерении напряжений прикосновения и токов в электроустановках должны быть установлены режимы и условия, создающие наибольшие значения напряжений прикосновения и токов, воздействующих на организм человека.

 

 

(справочное)

 

Термины и их пояснения

 

Термин

Пояснение

Напряжение прикосновения

По ГОСТ 12.1.009-76

Аварийный режим электроустановки

Работа неисправной электроустановки, при которой могут возникнуть опасные ситуации, приводящие к электротравмированию людей, взаимодействующих с электроустановкой

Бытовые электроустановки

Электроустановки, используемые в жилых, коммунальных и общественных зданиях всех типов, например, в кинотеатрах, кино, клубах, школах, детских садах, магазинах, больницах и т.п., с которыми могут взаимодействовать как взрослые, так и дети

Отпускающий ток

Электрический ток, не вызывающий при прохождении через тело человека непреодолимых судорожных сокращений мышц руки, в которой зажат проводник

 

(Измененная редакция, Изм. № 1).

 

 

Напряжение

— насколько безопасно 48 В постоянного тока?

Смерти от электрического тока не произойдет

48V считается «безопасным», и на то есть веские причины.

Во-первых, полное сопротивление человеческого тела при 50 В составляет около 45 кОм (хотя измерено на взрослых). В то время как дети в целом меньше и, следовательно, должны иметь немного более низкий импеданс, именно сопротивление кожи составляет 95% этого импеданса (внутренние жидкости организма являются довольно хорошими проводниками), поэтому размер не имеет значения для всех , а .

(Обратите внимание, насколько забавно импеданс тела, он быстро падает при повышении напряжения, при 240 В оно в 10-15 раз ниже!)

Далее, очевидно, электрический ток должен куда-то уходить. Ни замкнутой цепи, ни тока. Поэтому птиц, сидящих на суше, не жарят.

Эти 48 В составляют 48 В относительно земли. По всей вероятности, следующая ближайшая к «земле» вещь, с которой вы контактируете, — это «паркет / ламинат» или «напольная плитка» или что-то подобное, другими словами, сопротивление около бесконечности, текущий ноль.
Даже прикосновение к горячему проводу на 240 В имеет хороший шанс, что «не случится много плохого», если вы носите обувь и не стоите точно в луже воды (хотя по понятным причинам я бы не советовал испытывать удачу! ).

Предположим, что это наихудший случай: ребенок кладет палец на штырь заземления на розетке и сосет кабель PoE (выглядит съедобным, не правда ли!). Несмотря на все трудности, PSE неисправен или сильно не соответствует требованиям, и вместо подачи макс. 10,2 В / 4 мА по умолчанию он обеспечивает полное рабочее напряжение и неограниченный ток.Или требуется какой-то случайный образец, который ребенок случайно создал для правильного согласования, как бы то ни было.
Кроме того, по необъяснимой причине ток не замыкается на проводах данных (скорее всего, произойдет именно это, небольшая искра на языке ребенка, и ребенок в испуге уронит кабель).
Скажем так, на самом деле на проводе 40 В, и ток «решает» пройти через тело ребенка вопреки всем причинам и вопреки законам физики.

Кабель во рту устраняет один кожный барьер и, таким образом, уменьшает сопротивление тела примерно вдвое.Осталось 22,5 кОм. Для уверенности округлим до 20 кОм. Нет, знаете что, давайте будем возмутительными и скажем 10 кОм. 48 В / 10 кОм = 4,8 мА.
Которая … безвредна даже для переменного тока. Чтобы остановить сердце, требуется примерно в 8-10 раз больше переменного тока (с частотой в критическом диапазоне 50-60 Гц).

Кроме того, PoE не имеет переменного тока , это постоянный ток. Так что страшная фраза об остановке сердца даже не применима.

Конечно, постоянный ток в принципе может вызывать другие побочные эффекты, кроме остановки сердца (подумайте о хирургическом электрическом ноже или «электрическом стуле»), но, учитывая напряжения в двузначном диапазоне и токи в однозначном миллиамперном диапазоне, этого просто не произойдет (но даже если бы это было так, это были бы в первую очередь местные ожоги, а не угроза жизни).

Являются ли электрические ограждения серьезным риском для безопасности людей?

Функция электрического ограждения заключается в том, чтобы удерживать цель вне заданной области или удерживать цель в заданной области. Для этого электрическое ограждение должно использовать подаваемую мощность, чтобы выполнять функцию, которую его просят выполнять, самым безопасным из доступных способов.

Использование электричества сразу вызывает в воображении идею о том, что это опасно, прикосновение к электрическому забору оставляет яркое и болезненное воспоминание, а напряжения также высоки (+5000 вольт) по сравнению со стандартным сетевым электричеством (220 вольт), из-за этого большинство из них будет предполагаем, что риск для жизни и здоровья также должен быть высоким.На самом деле — это наоборот . Учтите, что сотни тысяч людей во всем мире «подвергаются» воздействию миллионов электрических ограждений каждый день — , однако электрические ограждения составляют (но не всегда являются причиной) менее одной серьезной травмы в год во всем мире. Сравните это с количеством ежегодных травм и смертей от воздействия тракторов, погрузчиков, лестниц, валов отбора мощности, пресс-подборщиков, косилок, комбайнов, быков, жеребцов, ружей, ножей и т. Д.Это не означает, что риска нет. Действительно, существует небольшой уровень риска. А с риском есть еще и ответственность перед владельцем забора.

Как достигается такой уровень безопасности?

Напряжение, передаваемое по лентам или проводам, высокое, но ток или усиление (в амперах) очень низкие. Разряд 220 или 110 вольт повредит, если ток или амперы одинаковы. Эти токи постоянны и имеют высокую силу тока (13 и 20 А соответственно) для питания машин и посуды.Плита может потреблять до 30 ампер.

Вторая проблема со стандартным сетевым электричеством заключается в том, что мышцы — человеческие и животные — напрягаются при контакте с любым типом тока. В этом случае нет возможности разорвать контакт. Вы просто физически привязаны к источнику, и, как бы вы ни старались, вы не можете отпустить — постоянное напряжение держит ваши мышцы заблокированными, а с высоким током — вы будете жариться.

Убивает

ампер. Напряжение привлекает ваше внимание — это больно.

Энерджайзеры для электрических заграждений выдают высокое напряжение (около 8000 вольт), что оставляет очень четкий мысленный отпечаток, который действительно привлекает внимание цели. Однако они также снижают смертельный ток до очень низкого значения около 120 миллиампер. (Это зависит от производителя). Это 120 тысяч ампер (нормальное напряжение в сети — 13 ампер). Он не должен убивать даже белку.

Этот выходной сигнал обеспечивается двумя способами, во-первых, путем высвобождения потока электронов из конденсатора в виде регулярных импульсов с интервалом примерно 1/300 секунды с интервалом примерно в секунду.Сила тока электрического заряда значительно снижена до значений в диапазоне 15 — 500 миллиампер. (Большинство устройств работают в диапазоне 100–150 мА) *. Сравните это с двумя другими сценариями.

  • Статическое электричество, когда вы касаетесь двери, около 30000 вольт при 0,5 миллиампер на 1/1000. секунды, неприятно, но недолго.
  • Электросеть. 220 вольт при 13 ампер и постоянное, неприятное и регулярно приводящее к смерти, есть много случаев, когда люди отключают провода, непосредственно подключенные к цепям 220 или 110 в, убивая как животных, так и людей.
  • Сварщик использует ток в 200 ампер, чтобы прожечь сталь.

В современных блоках питания ограждения используются схемы с низким импедансом, в которых конденсатор заряжается по твердотельной схеме. Если животное (или человек) соприкасается с забором с помощью изоленты или провода, заряд высвобождается тиристором. Это электронный компонент, который можно рассматривать как автоматический переключатель, поэтому подаваемое напряжение более контролируемо, а ударный импульс намного короче — обычно всего несколько миллисекунд.Энергия пульсирует через ленты, провода или другие проводники. Это означает, что раз в секунду в течение 1/300 секунды он посылает электрический импульс по линии.

Причина пульсирующего тока заключается в том, что при прикосновении к проводам возникает электрический ток — все, к чему прикасается, имеет шанс самоуничтожиться, потому что, когда животное касается электрического провода, оно вызывает сокращение мускулов у животного, подобное тому, которое люди ощущают мышечную судорогу, но это временное явление, и пораженное животное может уйти от источника электричества — ленты, веревки или проволоки забора.При непрерывной подаче тока, который вы получаете от электросети, это приводит к такому опасному эффекту захвата, что жертва не может высвободить источник тока. При пульсации электрического забора спазмы носят временный характер, и пострадавший может отступить от источника энергии. У животного это неприятное чувство будет ассоциироваться с прикосновением к забору, и ему не захочется прикасаться к нему снова в будущем.

Как работает энергозаборник.

Если ток не пульсирует (как у большинства электроприборов — фен, радио, тостер и т. Д.), То все, к чему он прикасается, будет продолжать подвергаться электрошоку до тех пор, пока не отключится электричество или что-то не отключит их.

Электрические ограждения с низким током и пульсирующим током являются безопасным продуктом. Это сила тока в электрическом заряде и постоянное соединение делают электричество опасным.

Одна проблема может возникнуть, если животное на какое-то время застрянет в заборе и не может выбраться.Это может быть из-за того, что животные с рогами, ежи, свернувшиеся в клубок, или по любой другой причине, они оказались в ловушке. Это зависит от разных животных и, к сожалению, может привести к смерти животного. К счастью, это случается очень редко, и за 30 лет работы с электрическими ограждениями я знаю 3 случая, когда было убито животное. По этой причине были разработаны блоки питания HoriSmart, способные распознавать то, что касается ограждения, обрабатывать его соответствующим образом, соответственно повышая безопасность.

Чего НЕЛЬЗЯ делать!

  • Никогда не приближайте голову к электрифицированному проводу. Случайный контакт головы или шеи может произойти при проталкивании датчика напряжения в почву или при проверке напряжения. При этом будьте очень осторожны, чтобы избежать контакта головы с проводами!
  • Никогда не позволяйте никому прикасаться к современному электрическому забору. Это не игра!

Что делать!

  • Проинструктируйте всех посетителей и детей никогда не прикасаться к электрическим ограждениям.
  • Законодательство, которое применяется в Европе к ограждениям, доступным для широкой публики, гласит, что международно признанный предупреждающий знак должен быть размещен в начале и конце забора и через каждые 50 метров.

Предупреждение: В 1991 году произошел несчастный случай со смертельным исходом, когда голова маленького ребенка коснулась электрифицированного забора, когда ребенок ползал по мокрой траве. Забор был установлен правильно и исправно функционировал. Энерджайзер был одобренным устройством.Поэтому мы настоятельно не рекомендуем позволять малышам приближаться к электрифицированным ограждениям. Кроме того, из-за этого и других инцидентов эксперты теперь предполагают, что контакт человека с помощью провода под напряжением к голове и шее может быть самой опасной точкой контакта. Мы призываем всех особенно избегать такого контакта с электрическими ограждениями.

* Эти цифры различаются у разных производителей.

Квартиры Footrot от Мюррея Болла.

Что такое электробезопасность, безопасное напряжение и ток для человеческого тела?

Если вы инженер-электрик или ваша работа связана с электричеством, вы должны знать основы электробезопасности .Потому что жизнь — очень важный предмет в мире. В этом посте я расскажу об электрической безопасности, которая действительно может спасти вас во время работы.

Электробезопасность:

Да, поток воды мы видим. Воздух это может чувствовать, но ток через проводник мы не видим. Вот почему проблема с электричеством. По этой причине очень важно знать, как и почему электричество становится опасным для человеческого организма.

Связано: 15 Важные процедуры и меры предосторожности в отношении электробезопасности

Ежегодно сотни или, может быть, тысячи людей гибнут и получают травмы электроэнергией .Многие аварии можно было бы предотвратить, если бы персонал просто знал, как и почему электричество опасно . Многие такие аварии вызывают остановку завода и приводят к гибели людей и материальному ущербу. Опасность поражения электрическим током становится причиной смерти и материального ущерба. Итак, нам нужно обратить внимание на эту смертельную опасность.

Причина поражения электрическим током:

Вот ситуация, при которой может произойти опасность поражения электрическим током

  1. Короткое замыкание
  2. Искра
  3. Искра
  4. мигающий
  5. Взрыв
  6. Свободное соединение
  7. Меньше размера Кабель

Теперь опасность поражения электрическим током можно разделить на две категории

    1. Опасность для человека

По потоку электроэнергии

    1. Опасность для богатства

Причины возгорания в результате электрического короткого замыкания
Причины взрыва / возгорания от электрической искры

Безопасное напряжение и ток для человеческого тела:

Физиологический эффект электрического тока можно предсказать с помощью диаграммы, показанной ниже.

Ток (А) Физиологический эффект
0.2 СМЕРТЬ (сердечная мышечная недостаточность)
0,1 Крайние затруднения дыхания, мышечный паралич, болезненные ощущения
0,001 Mid Sensation

Надеюсь, вам понравился этот пост. Если да, поделитесь с другими и сообщите им о влиянии опасности поражения электрическим током. В следующем посте я расскажу о текущем воздействии на человеческий организм.

Смертельный удар электрическим током: какое напряжение вызывает смерть?

Вопрос с подвохом.Само по себе напряжение — не единственный фактор, способствующий серьезности поражения электрическим током. Ток, обычно измеряемый в амперах, также является важной частью уравнения, наряду с другими второстепенными факторами.

Напряжение — это мера давления или силы электрической энергии, проходящей через проводник, в то время как ток — это, скорее, показатель скорости электрического потока. Это ток, проходящий через тело, сжимает сердце или вызывает его фибрилляцию, что может привести к смерти.

Так что вопрос действительно должен быть: Сколько тока нужно, чтобы кого-то убить?

Ответа очень мало. Сила тока всего 0,007 ампер (7 мА) через сердце в течение трех секунд достаточно, чтобы убить. Прохождение 0,1 ампер (100 мА) через тело почти наверняка приведет к летальному исходу.

Однако сила тока при поражении электрическим током определяется напряжением и сопротивлением цепи. Человеческое тело обладает высоким сопротивлением электрическому току, что означает, что без достаточного напряжения опасное количество тока не может протекать через тело и вызывать травмы или смерть.Как показывает практика, более пятидесяти вольт достаточно, чтобы пропустить через тело потенциально смертельный ток.

Другие факторы, которые могут определять степень поражения электрическим током, включают продолжительность удара и место его попадания в тело. Например, удар током, передаваемый от одной руки через грудь к другой руке, намного опаснее, чем удар между двумя пальцами ног.

Вот несколько примеров:

  • Удар статическим электричеством может составлять 20000 вольт или более, но при очень низком токе и на очень короткое время: безвредно
  • Аккумулятор 9 В находится под недостаточным напряжением для прохождения опасного уровня тока через тело: безвредный
  • Розетка 240 В переменного тока находится под опасным напряжением и более чем способна пропускать очень опасный ток: потенциально смертельный
  • Разряд молнии может быть миллиардом вольт и обеспечивать чрезвычайно высокий ток (около 30 000 ампер): потенциально смертельный

Низкое напряжение, но не низкий риск

Хорошая новость заключается в том, что электрический ток через низковольтные кабели, такие как телекоммуникации, системы безопасности или видео, недостаточно силен, чтобы вызвать смертельный удар электрическим током.Плохая новость заключается в том, что при установке низковольтного оборудования все еще существует реальная угроза безопасности.

«Даже на низковольтных установках подрядчикам легко обрести чувство самоуспокоенности, которое может вызвать ложное чувство безопасности», — сказал Майкл Джонстон, исполнительный директор по стандартам и безопасности Национальной ассоциации подрядчиков по электротехнике (NECA). «Вот почему важно, чтобы подрядчики соблюдали Национальный электротехнический кодекс (NEC) при выполнении любых работ по установке, в том числе низкого напряжения.”

Он отметил, что Глава 7, Особые условия, и Глава 8, Системы связи, включают темы, особенно относящиеся к низковольтным установкам. Джонстон предупредил, что подрядчикам, работающим с низковольтным оборудованием, следует опасаться вторичного поражения электрическим током.

«Даже установщики низкого напряжения могут по-прежнему испытывать тревожные удары, особенно если они работают в небезопасной среде, например, если они не заземлены или стоят на мокром месте», — сказал Джонстон. «Хотя маловероятно, что такие удары сами по себе приведут к серьезным травмам, они, безусловно, могут буквально вывести кого-то из равновесия, что могло бы представлять реальную угрозу безопасности, если бы установщик стоял на лестнице.

«Кроме того, дуга в системе низкого напряжения имеет такой же потенциал воспламенения взрывчатых материалов, как и дуга в системе с напряжением 120+ вольт. Глава 5 NEC содержит правила для установки электропроводки любого напряжения в опасном (классифицированном) месте, особенно в местах с высокой концентрацией легковоспламеняющихся или горючих жидкостей, легковоспламеняющихся или горючих паров, проводящей и горючей пыли и так далее ».

Не заходи в слепую

Фил Джейнвей, председатель комитета по кодам BICSI, сказал, что монтажники низкого напряжения должны знать, как распознать электрические кабели высокого напряжения и держаться подальше от них.BICSI — это профессиональная ассоциация, поддерживающая индустрию информационных транспортных систем (ИТС).

«Большинство низковольтных кабелей в какой-то момент перекрещиваются с устройствами, которые действительно проводят опасный электрический ток, такими как электронные переключатели, мультиплексирующее оборудование и системы PBX [частные телефонные станции]», — сказал Джейнвей. «Таким образом, при установке низкого напряжения никогда не возникает риска высокого напряжения. Из-за условий, в которых вы находитесь, при работе с электроникой почти всегда существует какой-либо риск высокого напряжения.

«Подрядчик должен убедиться, что его установщики знают, когда следует остановиться и привлечь к работе лицензированного и опытного электрика», — сказал он. «Важно, чтобы они осознавали, с чем работают. Не позволяйте им заходить вслепую. Попросите их использовать VOM [вольт-ом миллиамперметр], чтобы определить, работают ли они в среде с высоким или низким напряжением. Но если установщиков попросят установить цепь высокого напряжения на электронику, а они не будут квалифицированы, то их безопасность будет поставлена ​​под угрозу.”

Волоконная оптика

«Оптоволоконные кабели

— это низковольтная технология, которая предъявляет некоторые уникальные требования к безопасности при установке», — сказал Джон Джей, менеджер группы разработки приложений оптического волокна Corning Inc., Корнинг, штат Нью-Йорк.

Во-первых, установщикам, выполняющим заделку оптоволоконных кабелей в полевых условиях, необходимо тщательно отслеживать и обрабатывать неиспользуемые осколки стекла, которые были отсечены от кабеля для заделки в полевых условиях.

«Они похожи на занозы, только их намного труднее увидеть или удалить, особенно когда они попадают в ткань или переплетение», — сказал Джей.

Один из лучших способов закрепить оторванные осколки стекла — это использовать простую петлю из ленты, сказал он, добавив, что «так не будет пальцев, пытающихся поднять их или управлять ими».

Существуют также недорогие и эффективные коммерчески доступные устройства для удаления волоконно-оптических кабелей от различных производителей.

Безопасность глаз — важная проблема при работе с оптоволоконными кабелями. Осколки волокна в глазу могут быть как очень болезненными, так и опасными, поэтому важно, чтобы установщики носили защитные очки при работе с волокном и не касались лица, пока не вымыли руки.И никогда не стоит смотреть прямо в оптоволоконный кабель, через который может протекать лазерный ток, сказал Джей.

Он также предупреждает, что оптоволоконные разъемы следует содержать в чистоте, поскольку грязь на конце разъема может быстро окислиться при приеме сигнала, что создает как возгорание, так и риск нарушения нормальной работы. Установщик должен проявлять особую осторожность при использовании сварочного аппарата, который соединяет волокна вместе с помощью электрической дуги. Рабочие места для сварки должны быть сухими и чистыми, в них не должно быть легковоспламеняющихся паров, в том числе паров изопропилового спирта, который часто используется для очистки волоконной оптики.

Многое о безопасности

Reese Electric, многопрофильный подрядчик, базирующийся в Норт-Бенде, штат Орегон, понимает важность низкого напряжения и безопасности на рабочем месте. В 2008 году Риз занял первое место в конкурсе Associated General Contractors / Willis Construction Safety Excellence Award для специализированных подрядчиков, наработавших менее 100 000 рабочих часов. Неудивительно, что владелец Reese Electric Рэнди Рема много говорит о безопасности.

«Работы с низковольтным оборудованием сопряжены со многими из тех же опасностей, что и любая другая строительная отрасль», — сказал он.«Самое важное, что я могу сделать, — это дать моим сотрудникам возможность принимать решения, связанные с безопасностью, во время работы. Все мои бригады имеют право прекратить работу, если они окажутся в небезопасном состоянии. И наши ребята также должны знать, что руководство поддержит их в таких ситуациях, иначе вся идея расширения прав и возможностей сотрудников улетучится ».

Рема поделился множеством анекдотов о случаях, когда его экипажи поступали именно так, останавливая работу в небезопасных ситуациях, пока риски не были устранены.Было время, когда бригада обнаружила множество обнаженных игл для подкожных инъекций неизвестного происхождения, которые валялись в подвесном пространстве здания, куда бригады Reese Electric были направлены для выполнения работы. В другой раз команда наблюдала, как асбест выступает из изоляции трубы на чердаке старого школьного здания.

Его люди не были наказаны за прекращение работы в таких обстоятельствах; фактически, они были утверждены за то, что они принимали правильные решения.

«Возможно, они даже не рискуют», — сказал Рема.«Если мои ребята видят что-то небезопасное для другой компании или профессии, я хочу, чтобы они прекратили работу, пока ситуация не будет урегулирована. Если кто-то делает что-то небезопасное — мне все равно, кто это — скажите ему, чтобы он прекратил. При необходимости сообщите об этом генподрядчику. Но не продолжайте работать, если происходит что-то небезопасное ».

Рема объяснил отчасти свою мотивацию: «Однажды я был на рабочем месте, где получил серьезную травму. Производство всех участников этого проекта, а не только компании, попавшей в аварию, упало на две недели.Весь проект практически остановился ».

Рема также подчеркнул важность выявления опасностей и их устранения. Это включает в себя поиск и устранение опасностей, связанных со споткнувшись, принятие решения о безопасном перемещении оборудования и использование защиты от падения. Это также означает определение того, что делать, когда опасности не могут быть устранены.

«Умение отличать означает обучение и образование», — сказал он. «Вашим людям необходимо знать, что им делать и кому звонить, когда они попадают в ситуацию, которая им не по силам.”

Именно поэтому компания Reese Electric ежемесячно проводит обязательные собрания по технике безопасности для своих сотрудников. Примерно восемь месяцев в году Rema использует внешние ресурсы, такие как представители местного комитета по безопасности, консультанты по убыткам Ассоциированных генеральных подрядчиков или местный сотрудник Управления по охране труда (OSHA). Раз в год на собраниях основное внимание уделяется безопасному использованию тяжелого оборудования компании. А в оставшиеся месяцы они сосредотачиваются на текущих горячих темах, которым должны быть подвержены экипажи.

Рема не только пользуется услугами местного специалиста по соблюдению требований OSHA для ежемесячных встреч по безопасности.

«Мы обнаруживаем, что по мере того, как мы проводим с ним время, он привыкает к нам и видит нашу приверженность безопасности, а наши установщики привыкают к нему. Иногда мы проходим потенциально опасные места работы [с] сотрудником OSHA еще до того, как работа начнется. Все это помогает всем чувствовать себя комфортнее друг с другом, чтобы мы могли работать вместе, чтобы обеспечить безопасные рабочие места ».

Наконец, Рема считает, что признание сотрудников является важной частью стратегии безопасности компании.Для Reese Electric это означает ежегодный банкет с вручением наград в области безопасности для всех сотрудников компании.

«Но у нас не было бы их, если бы у нас началась череда несчастных случаев», — сказал Рема.

Хотя признание стоит компании денег, Рема сказал, что более низкие страховые расходы его компании помогают оплачивать их.

«И дело не только в деньгах», — сказал он. «Личные и финансовые затраты были бы ужасными, если бы наши показатели безопасности не находились под контролем. Все наши сотрудники действительно гордятся нашей репутацией в области безопасности и полученными наградами.И вы можете быть уверены, что никто не захочет быть первым, кто все испортит и пострадает ».

МУНЯН — писатель-фрилансер из Олате, штат Канзас, специализируется на технических и деловых текстах. С ним можно связаться на сайте www.russwrites.com.

Шоковые батареи — поражение электрическим током от батарей

Поскольку использование батарей с напряжением выше 300 В становится все более обычным явлением с ростом популярности электрических и гибридных электромобилей, существует опасность того, что широкая публика, столь привыкшая к относительно безвредным 12-вольтовым батареям, может недооценить связанные с этим опасности с тяговыми аккумуляторами более высокого напряжения.Поражение электрическим током составляет около 1% всех несчастных случаев со смертельным исходом, в основном среди людей, которым следует знать лучше.

На этой странице описаны опасности и некоторые меры предосторожности при работе с высоковольтными батареями.

Удар электрическим током

Физиолог может рассматривать тело как содержащее электрическую сеть, передающую крошечные нервные сигналы вокруг, позволяя нам делать все те важные дела, которые нам так нравятся, например, дышать, думать и двигаться.Его функция может быть серьезно нарушена из-за постороннего тока. Тело также содержит сеть каналов, транспортирующих кислород к мышцам и мозгу в соленом растворителе, называемом кровью, который, кстати, обеспечивает хорошую проводящую среду для электричества.

Однако для батареи корпус — это просто изолированный кожный мешок, содержащий электролит. См. Также нервных импульсов .

Несмотря на то, что напряжение часто используется в качестве индикатора опасности и подразумевается в первом абзаце, оно не является надежным индикатором степени тяжести поражения электрическим током.Наиболее важными показателями являются фактический ток, протекающий через тело, и его продолжительность, и даже они могут привести к ошибочным выводам, поскольку физиологические последствия зависят от маршрута, по которому ток проходит через тело. Ток, проходящий через сердце или мозг, несравненно более опасен, чем ток, проходящий через палец или ладонь, зажатую между выводами батареи. Продолжительный ток также нанесет больший ущерб, чем короткий импульс тока.

Физиологические последствия поражения электрическим током

В таблице ниже описаны некоторые эффекты постоянного электрического тока, проходящего через тело в течение одной секунды.

Важные примечания: Две таблицы на этой странице составлены из различных источников, и, хотя существует общее согласие между источниками по величине причин и следствий, фактические значения могут сильно отличаться. Очевидно, что нецелесообразно проводить тесты на людях, чтобы проверить уровни, при которых удары становятся фатальными, и некоторые данные получены на животных. Поэтому используемые значения являются средними или типичными значениями, которые следует использовать только в иллюстративных целях.

Опасные токи показаны в Красный .

Шокирующие эффекты

Ток (контакт 1 секунда)

Физиологический эффект

Менее 1 мА

Нет ощущений

1 мА

Порог ощущения.Ощущение покалывания

5 мА

Максимальный безопасный ток

8-15 мА

Легкий шок

Начало мышечного сокращения.

Нет потери мышечного контроля

15-20 мА

Болезненный шок

Устойчивое мышечное сокращение.

Не могу отпустить проводник

20-50 мА

Не могу дышать. Паралич грудных мышц

,00

Возможно со смертельным исходом

50-100 мА

Сильная боль

Нарушение дыхания

Фибрилляция желудочков

Вероятно со смертельным исходом — со смертельным исходом при продолжении

100-200 мА

Фибрилляция желудочков

Вероятно со смертельным исходом — Смертельно, если продолжение

Дыхательная функция продолжается

Более 200 мА

Устойчивые сокращения желудочков, сопровождающиеся нормальным сердечным ритмом (дефибрилляция)

Мышцы груди сжимают сердце и останавливают его на время разряда.Это также предотвращает фибрилляцию желудочков, повышая шансы на выживание, но в игру вступают и другие факторы.

Ожоги

Временный паралич дыхания.

Вероятно со смертельным исходом — со смертельным исходом при продолжении

Более 1 А

Сильные ожоги.

Внутренние органы сожжены.

Смерть

Выживаемость, если жизненно важные органы не на текущем пути — например, через палец

Примечания:

  • Низкое напряжение не означает малую опасность.

  • При прочих равных степень травмы пропорциональна продолжительности времени, в течение которого тело находится в цепи.

  • Согласно IEEE Std. 80, максимально безопасную продолжительность разряда можно определить по формуле

    Т = 0.116 / (E / R), где T — время в секундах, E — напряжение, а R — сопротивление человека (предполагается, что 1000 Ом).

    Для цепи на 120 В максимальная продолжительность разряда = 0,116 / (120 В / 1000) = 1 секунда
    Для цепи 240 В максимальная продолжительность разряда = 0,116 / (240 В / 1000) = 0,5 секунды

  • Чрезвычайно важно освободить пострадавшего от шока от как можно быстрее связаться с током.Разница в несколько секунд в запуске искусственного дыхания может означать для пострадавшего жизнь или смерть. Не сдавайтесь, если врач не объявил жертву мертвой.
  • Женщины более восприимчивы к электрическому току, чем мужчины
  • Меньшая масса тела увеличивает восприимчивость к электрическим токам
  • Разряд от постоянного тока с большей вероятностью остановит или остановит сердце жертвы.
  • Текущий диапазон от 100 до 200 мА особенно опасен, потому что он почти наверняка приведет к летальной фибрилляции желудочков, сотрясению сердца до бесполезного трепетания, а не регулярного биения.
  • Порог фибрилляции является функцией тока во времени. Например, фибрилляция произойдет при токе 500 мА в течение 0,2 секунды или 75 мА в течение 0,5 секунды.

  • Переменный ток более опасен, чем постоянный ток, вызывая более сильные мышечные сокращения. AC также с большей вероятностью вызовет фибрилляцию сердца жертвы, что является более опасным состоянием. Следовательно, безопасные рабочие пороги намного ниже для переменного напряжения.
  • Легче перезапустить остановленное сердце после устранения источника электрического шока, чем восстановить нормальный ритм биения фибриллирующего сердца.Сердце, которое находится в состоянии фибрилляции, не может быть восстановлено до нормального состояния с помощью закрытого массажа грудной клетки. Дефибрилляторы дают сердцу импульс постоянного тока, чтобы остановить фибрилляцию и позволить сердцу возобновить нормальный ритм.
  • Жертвы удара высоким напряжением обычно лучше реагируют на искусственное дыхание, чем жертвы удара низкого напряжения, вероятно, потому, что более высокое напряжение и ток зажимают сердце и, следовательно, предотвращают фибрилляцию. Шансы на выживание велики, если жертве уделить немедленное внимание.
  • Жертвы шока могут страдать от сердечного приступа в течение нескольких часов после шока. Опасность поражения электрическим током не исчезает после оказания немедленной медицинской помощи.
  • Не ждите, что вас защитит отключение при утечке на землю или детектор замыкания на землю (автоматический выключатель). Обычно они срабатывают при 15 ампер.

Потенциал шока

В то время как сила удара электрическим током в основном определяется током, на ток, в свою очередь, влияют многочисленные переменные, которые составляют сопротивление пути тока, что затрудняет прогнозирование тока, который будет течь при заданном напряжении.Двумя основными составляющими сопротивления являются сопротивление тела между точками контакта с электрической цепью и сопротивление контакта между телом и источником напряжения. Более подробно сопротивление тела зависит от длины проводящего пути через тело и веса тела. Сопротивление контакта зависит от того, является ли контакт влажным или сухим, от площади контакта, прочности захвата или касания электрического контакта и от наличия какой-либо другой изоляции на пути.Из-за такого большого различия в сопротивлении и продолжительности контакта, люди, как известно, пережили удары 40 кВ, в то время как другие были убиты менее чем 50 вольт

В следующей таблице показаны условия, которые могут привести к серьезному поражению электрическим током. Он дает сопротивление тела и контакта, связанное с различными условиями, и указывает ток, который будет течь при разных напряжениях. В таблице выше описаны последствия.

Токи от поражения электрическим током

Обстоятельства

Сопротивление

(Ом)

Ток мА

50 В

100 В

250В

500 В

Рука к земле (резиновые перчатки или подошва)

20,000,000

0.002

0,005

0,01

0,02

Рука к земле (Сухая рука, Кожаная подошва)

1,000,000

0,05

0.1

0,2

0,5

Сухая кожа

500 000

0,1

0,2

0.5

1

Легкое прикосновение (сухое)

500 000

0,1

0,2

0,5

1

Трос для захвата руки или металлический инструмент (сухой)

20 000

2.5

5

12,5

25

Легкое прикосновение (влажное)

10 000

5

10

25

50

Рука к земле (Мокрая рука, влажная кожаная подошва)

10 000

5

10

25

50

Трос для захвата руки или металлический инструмент (мокрый)

5,000

10

20

50

100

Из рук в руки (влажный)

1500

33.3

66,6

166

333

Мокрая кожа

1000

50

100

250

500

По всему телу человека

1000

50

100

250

500

Из рук в руки, из рук в руки (без кожи)

500

100

200

500

1000

Между ушами, на пальце (без кожи)

100

500

1000

2500

5000

Прокол кожи с порезами, ссадинами или ожогами от самого электрического тока

Нет сопротивления

Очень высокий

Очень высокий

Очень высокий

Очень высокий

Примечания:

  • Кожа — самый важный изолятор.
  • Сопротивление контактов сильно различается.
  • Работа с небольшими ранениями рук серьезно увеличивает риск поражения электрическим током.
  • После того, как был инициирован разряд, электрический ожог может проткнуть кожу и увеличить ток разряда.
  • Кольца, браслеты и другие украшения уменьшают контактное сопротивление с телом и увеличивают вероятность поражения электрическим током.
  • Используйте только одну руку (держа одну руку в кармане), пока работа с цепями высокого напряжения позволяет избежать риска того, что тело станет частью цепи.
  • Риски можно минимизировать, используя изолированные ручные инструменты (плоскогубцы, отвертки, гаечные ключи и т. Д.), А также надев резиновые перчатки и обувь.

Проектирование схем автомобильных аккумуляторов

Стандартные цепи автомобильной аккумуляторной батареи на 12 и 24 вольт полагаются на шасси транспортного средства в качестве заземляющего возвратного контура.Эта практика приемлема, когда не используются смертельные напряжения, но не для высоковольтных батарей, используемых в электромобилях и HEV, поскольку кто-то, кто работает с батареей, может легко стать проводником между любыми открытыми клеммами высокого напряжения (на батарее, двигателе и его контроллер) и шасси.

Поэтому в цепях аккумуляторных батарей

EV и HEV следует использовать изолированные аккумуляторные шины как для положительной, так и для отрицательной стороны батареи.

Это также важная функция безопасности при эксплуатации, так как случайное отключение изоляции может подвергнуть водителя или аварийные службы воздействию опасного напряжения или вызвать опасное короткое замыкание аккумулятора.В случае нарушения изоляции или случайного повреждения система контроля замыкания на землю, которая обнаруживает утечку тока из аккумулятора или инерционные переключатели, которые обнаруживают замедление высокого ускорения из-за аварии, должна автоматически отключать аккумулятор.

Методы безопасной работы

Существует множество опубликованных стандартов безопасной работы в цепях переменного тока, включая стандарты HSE в Великобритании и OSHA в США.См. Раздел «Стандарты».

Ниже приведены дополнительные рекомендации, относящиеся к работе с цепями батарей.

  • Обычный порог безопасного рабочего напряжения для работы от аккумуляторов составляет 50 вольт постоянного тока.
  • Отсоедините аккумулятор от нагрузки с очевидным, видимым отключением, а не просто выключателем, положение которого можно не заметить.
  • Не работайте с полностью заряженным аккумулятором.По возможности позаботьтесь о том, чтобы батареи были разряжены.
  • С помощью вольтметра перед началом работы проверьте, заряжен ли аккумулятор.
  • С помощью прибора для проверки целостности цепи проверьте отсутствие соединения между клеммами аккумуляторной батареи и оборудованием или шасси автомобиля (см. Выше).
  • Убедитесь, что все конденсаторы, связанные с цепью батареи, разряжены.
  • По возможности закрывайте открытые клеммы, чтобы не прикасаться к ним и не ронять на них инструменты.
  • При начале работы с предположительно обесточенной цепью сначала коснитесь тыльной стороной одной руки, чтобы в случае удара током мышечная реакция не заставила руку схватить проводника.
  • См. Также примечания выше.

Относитесь к мощным аккумуляторным батареям с таким же уважением, как и к сетевым напряжениям переменного тока.

См. Также Короткие замыкания и их последствия

История

Что более опасно для человеческого организма: переменный или постоянный ток и напряжение?

Воздействие обоих на человеческий организм различается, но один из них более опасен, чем другой

Если вы работаете с электронными продуктами, то, вероятно, знакомы с переменным током (AC) и постоянным током (DC), а также различиями между ними.

Менее известно об этих двух токах, какое воздействие они оказывают на человеческое тело, и какое из них более опасно.


Что бы ни случилось, контакт переменного или постоянного тока с человеческим телом может быть опасным. Однако фактический эффект варьируется, так как он зависит от нескольких различных факторов, включая количество подаваемого тока, продолжительность контакта с телом, путь тока, приложенное напряжение и импеданс самого тела.

Из всего вышесказанного, если дело доходит до одного или другого, переменный ток обычно можно рассматривать как более опасный из двух токов — вот почему:

1) Для начала, чтобы оба тока имели одинаковое воздействие на человеческое тело, величина постоянного тока постоянной силы должна быть в два-четыре раза больше, чем переменного тока; то есть требуется больше постоянного тока, чтобы вызвать такое же количество физических повреждений, как и переменный ток. Это связано с тем, что воздействие токов на тело является прямым результатом возбуждающих воздействий его величины, в частности, фактического включения и отключения самого тока.Такие возбуждающие действия включают стимуляцию нервов / мышц, индукцию сердечной фибрилляции предсердий или желудочков и многое другое.

Чтобы постоянный ток оказывал на человеческое тело такое же воздействие, как переменный ток, его поток постоянной силы должен быть в два-четыре раза больше, чем поток переменного тока.

2) Когда происходит смерть от поражения электрическим током, это обычно происходит из-за фибрилляции желудочков, и вероятность того, что человек пострадает от такого рода смертельной травмы, намного выше при контакте с переменным током, чем с постоянным током, потому что порог фибрилляции желудочков в организме человека при постоянном токе в несколько раз выше, чем для переменного тока.

3) Вообще говоря, сопротивление человеческого тела выше для постоянного тока и уменьшается только при увеличении частоты. Таким образом, опасность поражения электрическим током при контакте с постоянным током меньше, чем с переменным током.

4) Легче отпустить / удалить контакт с «токоведущими» частями в случае постоянного тока, чем переменного тока. Это противоречит распространенному мнению о том, что, поскольку чередующиеся циклы переменного тока проходят через ноль, человеку дается достаточно времени, чтобы отвести конечность / тело от самой части, тогда как при постоянном протекании постоянного тока возникает нет частотных колебаний, которые дают человеку кратковременный момент, чтобы оторвать свое тело.Основание для этого аргумента может быть взято из эксперимента «отпускание», о котором сообщалось в той же вышеупомянутой публикации 60479 IEC. в руке испытуемого; тока было достаточно, чтобы человек не мог разжать руку и уронить электрод.

Не вдаваясь во все детали фактического эксперимента, можно было сделать вывод, что испытуемым было легче освободить электрод, когда подавали постоянный ток, а не переменный ток.

Добавить комментарий

Ваш адрес email не будет опубликован. Обязательные поля помечены *